Você está na página 1de 105

Foundation Briefs

Advanced Level Sept/Oct 2013 Brief

Resolved: In a democracy, voting ought to be compulsory.

Sept/Oct 2013

Table of Contents

Table of Contents
Table of Contents .................................................................................................................................................... 1 The Structure of a Foundation Brief ................................................................................................................... 7 Defend Your Source ............................................................................................................................................... 8 Organizations ...................................................................................................................................................... 8 Authors.............................................................................................................................................................. 12 Topic Analysis One............................................................................................................................................... 15 Topic Analysis Two .............................................................................................................................................. 17 Aff Evidence ......................................................................................................................................................... 19 Compulsory voting increases turnout ............................................................................................................... 20 Voter turnout is higher in Australia than in voluntary voting democracies. RMF ................................... 20 Success of Mandatory Voting AMS ......................................................................................................... 21 Places emphasis on democratic procedure........................................................................................................ 22 Emphasizing democratic procedure allows the votes to be counted, rather than promoting an elite aristocracy that decides what is democratic. RMF ................................................................................ 22 Compulsory voting upholds the public good of voting in a democracy, DAT ......................................... 23 CV upholds democratic ideals, e.g. more clean and equitable campaigning, DAT ................................. 24 CV encourages voting from marginalized groups in society JMR ........................................................... 25 CV Ensures Better and more Accurate Representation JMR ................................................................... 25 Enhances election process ................................................................................................................................ 26 Guaranteed turnout means candidates focus on right issues during campaign. RMF .............................. 26 CV encourages candidates to engage with those Alienated in the Political System JMR........................ 26 Turnout inequality harms democracy ............................................................................................................... 27 Low turnout harms legitimacy of government. RMF ............................................................................... 27 Voluntary turnout is skewed toward the elite. RMF ................................................................................. 27 Low turnout harms the socioeconomically disadvantaged. RMF ............................................................. 28 Political participation places positive pressures on democratic government, and vice versa DAT ......... 28 Benefits of Compulsory Voting Outweigh Disadvantages AMS ............................................................. 29 Unequal Representation AMS .................................................................................................................. 29

foundationbriefs.com

Page 1 of 104

Sept/Oct 2013 Table of Contents Inequality Problems in UK AMS .............................................................................................................. 30 Problems with Unequal Participation AMS .............................................................................................. 30 Unequal turnout reinforces gaps in privilege, DAT 9/9 11:39pm ............................................................ 31 Mandatory voting is rooted in the concept of universal suffrage, DAT 9/10 12:08am ............................ 32 Law of Dispersion AMS ........................................................................................................................... 32 Compulsory voting brings the underrepresented to the voting booth. RMF ............................................ 33 Compulsory voting can eliminate voting inequalities, DAT .................................................................... 33 Trend of Increasing Voter Inequality JMR ............................................................................................... 33 Democracies must remain majoritarian, DAT .......................................................................................... 34 High Voter Turnout Creates Stability JMR .............................................................................................. 34 CV decreases income inequality ....................................................................................................................... 35 Summary of study: Repealing compulsory voting in Venezuela increased income inequality. RMF ..... 35 Theoretical background on why this happens: differences in voting costs/benefits. RMF ...................... 35 Circumstances of the study. RMF ............................................................................................................. 36 Removal of CV increased income inequality in Venezuela. RMF ........................................................... 36 Eliminating CV Increased Voter Inequality JMR ..................................................................................... 37 Success Examples ............................................................................................................................................. 38 Success in Belgium AMS ......................................................................................................................... 38 Queensland, Australia Success AMS ........................................................................................................ 38 Australias Success AMS .......................................................................................................................... 39 Low Turnout ..................................................................................................................................................... 40 Expanding the Vote is Necessary AMS .................................................................................................... 40 Low turnout undermines the idea of representative government, DAT ................................................... 40 Mandatory Voting Encourages Moderacy ........................................................................................................ 41 More Voting Tempers Politics AMS ........................................................................................................ 41 Political Parties Encourage Divisiveness by Seeking Greater Turnout AMS .......................................... 41 Prevents Free-Riding ........................................................................................................................................ 42 Democracy is a Public GoodCV Prevents Freeriding JRM .................................................................. 42 CV shifts the position of the median voter JMR ................................................................................... 43

foundationbriefs.com

Page 2 of 104

Sept/Oct 2013 Table of Contents Voluntary voting is less representative of the median voter JMR ............................................................ 43 Neg Evidence ........................................................................................................................................................ 44 Infringement on Personal Liberty ..................................................................................................................... 45 Overview of compulsory voting as an infringement of liberty. RMF ...................................................... 45 Mandatory Attendance at polls is still Mandatory Voting AMS .............................................................. 45 A forced vote can contradict notions of self-interest and personal liberty, DAT 9/9 11:48pm ................ 46 Punishment is Undemocratic AMS ........................................................................................................... 47 Possibility of Jail Time for Non-Voters is Undemocratic, JMR............................................................... 47 Abstention a crucial act in democracy .............................................................................................................. 48 Compulsory voting forces citizens into an unfair process which is potentially undemocratic, DAT ...... 48 Abstention underlines the power of the individual over government. RMF ............................................ 49 Abstention is a Valuable and Powerful Political Tool , JMR ................................................................... 49 Voting not a duty .............................................................................................................................................. 50 The value of voting can be reasonably contested. RMF ........................................................................... 50 Living in a liberal democracy allows citizens to choose to avoid politics JMR ....................................... 50 Voting is expressive, not decisive; therefore not a duty ............................................................................... 51 The chance that a cast vote decides the election result is effectively zero. RMF ..................................... 51 Voting is an expressive act, showing support for a candidate or issue. RMF .......................................... 51 Democracy is CompetitiveNo Duty to Vote ............................................................................................. 52 Costs of Mandatory Voting............................................................................................................................... 53 Mandatory voting artificially imposes disproportional costs, DAT ......................................................... 53 Costs are not Trivial AMS..................................................................................................................... 54 Voting against own interests/ideology .............................................................................................................. 55 Uneducated voters who are forced to vote end up voting against own interests. RMF ............................ 55 People cannot be expected to vote in an objective or well-informed manner, DAT ................................ 55 A dearth of informed voters turns elections into exercises of random probability, DAT ........................ 56 Citizen has a Responsibility Not to Vote Poorly JMR ............................................................................. 56 Compulsory voting does not change outcome .................................................................................................. 57 Removing compulsory voting does not change the outcome of an election. RMF .................................. 57

foundationbriefs.com

Page 3 of 104

Sept/Oct 2013 Table of Contents The underlying problems and inequalities of the voting process are not eliminated, DAT ..................... 58 Increase in Voter Turnout due to CV would not make a difference in Elections, JMR ........................... 59 Undermining the Legitimacy of Voting............................................................................................................ 60 Compulsory voting institutionalizes complacency with flaws in the electoral process, DAT ................. 60 Universal Voting can pollute the democratic system JMR ....................................................................... 61 Mandatory Voting would Decrease Quality of System .................................................................................... 62 Those who Do Not Want to Vote Do Not Make Good Voters AMS ....................................................... 62 Random Voting delegitimizes a democracy JMR .................................................................................... 62 Compulsory does not address the problems of the voting process, DAT ................................................. 63 Under a democracy, the responsibility of voting is different from the requirement to vote, DAT .......... 64 Disenfranchisement as a Punishment Undermines Democracy JMR ....................................................... 65 Compulsory Voting vs. Compulsory Turnout .................................................................................................. 66 Aff Counters.......................................................................................................................................................... 67 Alternatives do not address the problem entirely ............................................................................................. 68 Even the full confluence of alternatives is not as effective as compulsion, DAT .................................... 68 Political Parties do not Help Minorities Express Views ................................................................................... 69 The Problem with Political Parties AMS .................................................................................................. 69 Interests of Political Parties Conflict with Interests of Democracy AMS ................................................ 69 CV does change election outcomes .................................................................................................................. 70 Methodology: author compares election results before and after CV across Australian states. ............... 70 CV changed policy attitudes as well as policy itself. RMF ...................................................................... 70 Compulsory voting significantly affected the share of votes going to the Labor Party. RMF ................. 71 Other phenomena are unlikely to have caused this change. RMF ............................................................ 71 Unequal Turnout Equates to Unequal Influence JMR .............................................................................. 72 It Does Not Threaten Personal Liberty ............................................................................................................. 73 JS Mill, father of liberalism, saw participation as necessary for successful governance. RMF ............... 73 Concept of equal liberty supports CV. RMF ............................................................................................ 73 The liberty of the least privileged must be guaranteed, CV does this by ensuring they have a voice in elections. RMF .......................................................................................................................................... 74 The Meaning of Individual Liberty AMS ............................................................................................. 74

foundationbriefs.com

Page 4 of 104

Sept/Oct 2013 Table of Contents Abstract compulsion is enough to spur turnout and political involvement, DAT 9/10 12:03am ............. 75 The right to vote does not imply a right not to vote, DAT ....................................................................... 76 Liberalism allows for civic duties. RMF .................................................................................................. 76 Liberty cannot be achieved in democracy without popular sovereignty. RMF ........................................ 77 Abstention still exists under compulsory voting............................................................................................... 78 Blank ballots are common in countries with CV. RMF............................................................................ 78 In practice, all CV laws allow for casting a non-vote. RMF .................................................................... 78 Despite being able to opt out of voting, CV still remains important to turnout and attitudes. RMF .... 79 Abstention as a form of protest can be effectively carried out under CV, DAT ...................................... 79 Other forms of protest do not have to be problematic, DAT .................................................................... 80 Better Alternatives to Abstention under Compulsory Voting Rules ................................................................ 81 The Right to Withdraw Democratic Legitimacy JMR .............................................................................. 81 CV Protects the Right to Say No JMR...................................................................................................... 81 Neg Counters ........................................................................................................................................................ 82 CV does not increase political knowledge ........................................................................................................ 83 Multiple studies have found no link between CV and increased political knowledge. RMF ................... 83 Experimental tests on CV find no increase in knowledge or effort. RMF ............................................... 84 CV does not Increase Voter Interest, JMR ............................................................................................... 85 CV does not Increase Voter Knowledge JMR .......................................................................................... 86 There is no Cure to Apathy ............................................................................................................................... 87 Why More Voting is Not Always Good AMS ......................................................................................... 87 Apathy is inherent to mass democratic voting systems, DAT .................................................................. 87 Consensus is not a democratic value. ............................................................................................................... 88 Politics Cant be Moderated AMS ........................................................................................................ 88 True majoritarian rule and democratic legitimacy are not intertwined, DAT .......................................... 88 Democracy does not require true majority rule, which is assumed under CV, DAT ............................... 89 Response to Jury Duty Comparison.................................................................................................................. 90 Jury Duty and Voting Duty Serve Different Purposes AMS .................................................................... 90 None of the above option not satisfactory ..................................................................................................... 91

foundationbriefs.com

Page 5 of 104

Sept/Oct 2013 Table of Contents The act itself of not casting any ballot is needed, not just a none of the above option. RMF............... 91 There are Other Options ................................................................................................................................... 92 Practical Options AMS ............................................................................................................................. 92 The Power of Social Pressure AMS.......................................................................................................... 93 Give people a stake in their local government AMS ................................................................................ 94 The Power of Peer Pressure AMS ............................................................................................................ 94 Voting Ambience Helps AMS .................................................................................................................. 95 Modernize Voting Systems AMS ............................................................................................................. 95 Success of Online Voting Systems AMS.................................................................................................. 96 Can Decrease the Frequency of Elections to positively Influence Turnout, JMR .................................... 97 Automatic Registration Can Increase Voter Turnout JMR....................................................................... 97 Will not Help Disadvantaged Groups ............................................................................................................... 98 The Disadvantaged Tend to Vote Against Their Own Interests AMS ..................................................... 98 Cases ..................................................................................................................................................................... 99 Aff Case .......................................................................................................................................................... 100 Introduction: ................................................................................................................................................ 100 Contention One: Compulsory voting addresses the unfairness of socioeconomic divides effects on voting patterns ........................................................................................................................................................ 100 Contention Two: Compulsory voting is an equalizer for voices of resistance ........................................... 101 Contention Three: People deserve a fair choice ......................................................................................... 101 Conclusion .................................................................................................................................................. 102 Neg Case ......................................................................................................................................................... 103 Introduction: ................................................................................................................................................ 103 Contention One: Compulsory voting does not increase voter knowledge. ................................................. 103 Contention Two: People who are forced to vote tend to vote against their own interests. ........................ 104 Contention Three: Alternative methods of increasing voter turnout exist. ................................................. 104 Conclusion .................................................................................................................................................. 104

foundationbriefs.com

Page 6 of 104

Sept/Oct 2013

Outline of Brief

The Structure of a Foundation Brief


Topic Analysis
This is a general reflection on the resolution. It will provide to you an impression of the topic at hand, challenges you will face while debating, and a picture of where we see the debate headed.

Framework
Often times, the most important part of the debate is to actually win before the debate begins. With this section, we will set you up for such a feat. With unique analysis on how to lay the conditions for victory, you will be guaranteed to begin battle already with an advantage.

Strategy Sections
Foundation Briefs is committed to making sure you understand the evidence provided to you. We will never simply throw quotes at you and hope you can understand what we are trying to imply. That is where the Strategy Section comes in. At the beginning of all major sections (i.e. the section in the brief regarding alQaeda) there will appear a small section of original Foundation Briefs analysis to tell you how we see the evidence being used, what rhetoric will please the judge and which counterarguments to be prepared for. Important note: Webpages and online articles that are long and continuous will always be cited as page one (1)

foundationbriefs.com

Page 7 of 104

Sept/Oct 2013

Defend Your Source: Organizations

Defend Your Source


Organizations
The American Spectator
The American Spectator is a known U.S. monthly magazine covering news and politics, edited by R. Emmett Tyrrell Jr. and published by the non-profit American Spectator Foundation. The magazine funds political research for the betterment of society overall.

CBC News
CBC News is the division of the Canadian Broadcasting Corporation responsible for the news gathering and production of news programs on CBC Television, Radio and online services. Founded in 1941, CBC News is the largest news broadcaster in Canada with local, regional and national broadcasts and stations.

Center for European Policy Studies


Founded in Brussels in 1983, the Centre for European Policy Studies (CEPS) is among the most experienced and authoritative think tanks operating in the European Union today. CEPS serves as a leading forum for debate on EU affairs.

Federal Law Review


Published since 1964 by the ANU College of Law; the Federal Law Review ("FLR") is recognised as one of Australias leading academic law journals. The FLR specialises in matters of federal law. The FLR also analyses legal and policy questions relating to the nature of federalism, both in Australia and elsewhere.

The Guardian
Founded in 1821 The Guardian is a British National daily newspaper. Its combined print and online editions reach nearly 9 million readers, and it has been awarded National Newspaper of the Year many times, most recently in 2011.

foundationbriefs.com

Page 8 of 104

Sept/Oct 2013

Defend Your Source: Organizations

Guido Carli Free International University for Social Studies (LUISS)


LUISS is a private university founded in 1974 in Rome, Italy. It is an independent university set up by a consortium of private and public companies that reorganized a pre-existing institution. The University was ranked the first for Law Faculty and for international and diplomatic studies in an Italian university ranking in 2012.

Forbes
Forbes is an American business magazine owned by Forbes, Inc. Published biweekly, it features original articles on finance, industry, investing, and marketing topics. Forbes also reports on related subjects such as technology, communications, science, and law. Its headquarters are in New York City.

Harvard Law Review


The Harvard Law Review is the universitys journal of legal scholarship. The Review publishes articles by professors, judges, and practitioners and solicits reviews of important recent books from recognized experts. All articleseven those by the most respected authoritiesare subjected to a rigorous editorial process designed to sharpen and strengthen substance and tone.

Institute for Public Policy Research


The Institute for Public Policy Research is an independent charitable organization with more than 40 staff members, paid interns and visiting fellows. IPPR seeks to assist all those who want to create a society where every citizen lives a decent and fulfilled life, in reciprocal relationships with the people they care about.

Institute of Public Affairs


The Institute of Public Affairs is an independent, non-profit public policy think tank, dedicated to preserving and strengthening the foundations of economic and political freedom. The IPA also publishes the IPA Review, Australia's longest running political magazine. In 2008, the IPA Review was awarded the Sir Anthony Fisher Memorial Award for best magazine.

Inter-American Development Bank (IADB)


The IADB is the largest source of development financing for Latin America and the Caribbean. It was established in 1959 and supports Latin American and Caribbean economic development, social development and regional integration by lending to government and government agencies, including State corporations.

foundationbriefs.com

Page 9 of 104

Sept/Oct 2013

Defend Your Source: Organizations

International Institute for Democracy and Electoral Assistance (IDEA)


IDEA is an intergovernmental organization based in Stockholm, which supports sustainable democracy around the world. It was established in 1995 by an international agreement registered with the United Nations. Its main aim is to assist countries in developing democratic institutions.

The Max Planck Institute for the Study of Societies


The Max Planck Institute for the Study of Societies conducts advanced basic research on the governance of modern societies. It aims to develop an empirically based theory of the social and political foundations of modern economies by investigating the interrelation between economic, social and political action.

The National Interest


The National Interest is an American bi-monthly international affairs magazine published by the Center for the National Interest. It was founded in 1985.

New Matilda
New Matilda is an independent news source for analysis covering politics, consumerism, international affairs, and culture.

The New York Times


The New York Times is an American daily newspaper, founded and continuously published in New York City since September 18, 1851. It has won 112 Pulitzer Prizes, more than any other news organization.

Pacific Standard Magazine


Pacific Standard is an arm of the nonprofit Miller-McCune Center for Research, Media and Public Policy, we are supported by a combination of grants and advertising revenue. The Magazine aims to foster global conversation on important issues through research.

Political Research Quarterly


foundationbriefs.com Page 10 of 104

Sept/Oct 2013 Defend Your Source: Organizations Political Research Quarterly (PRQ), a peer-reviewed quarterly, publishes original research in all areas of political science. One of the top ranked journals in the field, PRQ publishes scholarly research of exceptionally high merit that covers the entire range of topics and subjects in political science. Goals include promoting diversity in the field and providing a meaningful arena for discussions about leading research and scholarship.

Quadrant Magazine
Quadrant magazine is the leading general intellectual journal of ideas, literature, poetry and historical and political debate published in Australia.

Time
Time is an American weekly news magazine published in New York City. It was founded in 1923 and for decades dominated by Henry Luce, who built a highly profitable stable of magazines. Time has the world's largest circulation for a weekly news magazine, and has a readership of 25 million, 20 million of which are in the US.

The Week
The various editions of The Week magazine provide perspectives of the week's news and editorial commentary from global media to provide readers with multiple political viewpoints. In addition to news and opinion, the magazine also covers science, business and the arts.

University of Adelaide
The Discipline of History at the University of Adelaide has a long and rich tradition of scholarly and teaching excellence. Its reputation has been built since the early 20th century by a succession of eminent historians. The universitys staff continue to build on this foundation, and have gained national and international recognition for their research and writing in British and European, Australian, Southeast Asian and American history.

foundationbriefs.com

Page 11 of 104

Sept/Oct 2013

Definitions

Authors
Jason Brennan
Jason Brennan is Assistant Professor of Ethics, Economics, and Public Policy at Georgetown University. He is the author of Libertarianism: What Everyone Needs to Know (Oxford University Press, 2012), The Ethics of Voting (Princeton University Press, 2011), and, with David Schmidtz, A Brief History of Liberty (Wiley-Blackwell, 2010). His research concerns democratic theory, government failure, civic virtue, and government legitimacy.

Anthony Downs
Anthony Downs is an economist specializing in public policy and public administration. Since 1977 he is a senior fellow at the Brookings Institution in Washington D.C.

Bart Engelen
Bart Engelen is a postdoctoral researcher at the Centre for Economics and Ethics.

Alvin Goldman
Alvin Ira Goldman is an American professor of Philosophy at Rutgers University in New Jersey. He previously taught at the University of Michigan and at the University of Arizona.

Nathan Hanna
Nathan Hanna is an Assistant Professor of Philosophy at Drexel University. He previously taught at Lawrence University in Appleton, Wisconson. He received his Ph.D. in Philosophy from Syracuse University in 2008. His research focuses on ethics, philosophy of law, and political philosophy.

Keith Jakee
Keith Jakee holds a Ph.D. in economics from George Mason University. Dr. Jakee was a senior lecturer at Royal Melbourne Institute of Technology (Melbourne, Australia) before joining the Wilkes Honors College. Before that he was a lecturer in the Department of Economics at Monash University (Melbourne). He was also a postdoctoral research scholar at the City University of Stockholm, Sweden, and visiting lecturer at University College Cork, in Ireland.

foundationbriefs.com

Page 12 of 104

Sept/Oct 2013

Definitions

Annabelle Lever
Annabelle lever is Associate Professor of normative political theory at the University of Geneva. Her research focuses on the nature and justification of privacy, the ethics of voting, democracy and judicial review and the justification of intellectual property.

Jonathan Levine
Is a former lecturer of American studies at Tsinghua University. He is currently a freelance journalist living in Hong Kong. His work has been featured in The New York Times, CNN, and the Atlantic.

Arend Lijphart
Arend Lijphart is a well- known political scientist specializing in election, voting systems, and democratic systems. He studied Political Science at Yale University and received his PhD in 193. Currently he is Research Professor Emeritus of Political Science at the University of California, San Diego.

Stephen Macedo
Stephen Macedo is Laurance S. Rockefeller Professor of Politics and the University Center for Human Values at Princeton University. He writes and teaches on political theory, ethics, public policy, and law, especially on topics related to liberalism, democracy, and citizenship

Piero Moraro
Dr. Piero Moraro is a moral and political philosopher, working mainly in the area of democracy and political obligation. He holds a Ph.D. in Philosophy from Stifling University and is a Lecturer in Justice Studies at Charles Sturt University.

Dr. Armin Schfer


Is a fellow at the Hanse Institute for Advanced Studies in Delmenhorst. He has been awarded the Otto Hahn Medal from the Max Planck Society for the Dissertation Thesis.

foundationbriefs.com

Page 13 of 104

Sept/Oct 2013

Definitions

Sidney Verba
Sidney Verba is The Carl H. Pforzheimer University Professor Emeritus and Research Professor of Government as Harvard University, where he taught for thirty-five years. At Harvard, he was also chair of the Department of Government, Associate Dean of the Faculty for Undergraduate Education, and Associate Provost, among several other senior administrative posts

foundationbriefs.com

Page 14 of 104

Sept/Oct 2013

Topic Analysis One

Topic Analysis One


In a democracy, voting ought to be compulsory. Thats eight wordsa short resolution. Scope it out with some quick searching, and youll likely see two basic camps: the voting is a civic duty group, and the not voting is an individual right groupessentially a stock aff and neg set of arguments. As you will see from the rest of this brief, we have tried to both dig into the substance of these two arguments and, more significantly, rigorously pursue other avenues of argumentation. It is, of course, up to you to decide what extra research to do, what ideas to give extra thought, etc. But lets pick this apart a bit to see if we can open some options aside from the stock approaches. Framework Analysis Debates on this resolution will see plenty of variations on the above aff and neg arguments. It could even be possible to flip them around (e.g. protest is also a civic duty; not voting is a form of protest). This means getting a grip on the resolutionits nuances, its limitationsis what will really kick-start some good thought into this topic (and hopefully some winning cases to boot). Heres what came to mind working my way through this resolution and its material. Respect ought The use of ought indicates a moral obligation, and the resolution needs to be treated from that viewpoint. Here is what I mean: there is a wealth of information out there about compulsory voting. Data abounds on voter turnout rates. Australia is practically a decades-long case study in the practice. But again, we stumble back to ought. This debate cannot turn into war of practicality. The idea of moral obligation is that while practicality can potentially be an important consideration, it is not the primary consideration. This is about what is morally right, even if what is morally right has practical implications. There are plenty of philosophical angles to look at this throughKants categorical imperative (invert the requirement to vote, see what the implications are), the social contract (Hobbes, Locke, or Rousseau, among others, seem applicable here), J.S. Mills individual liberty, etc. While the resolution certainly is not an invitation to name-drop political philosophers, understanding the political philosophy at the heart of modern democratic systems is what will create a genuine understanding of this topic. There is no actor That is to say, there is no one making voting compulsory. The resolution does not clarify who ought to value the compulsion of voting; it only states what compulsory voting is in respect to (democracy). We can therefore safely assume that the resolution is talking about what we as a people should value. Ive already mentioned some philosophical perspectives that can help guide a framework to this resolution. Just keep in mind that they should be used with knowledge of who the actor isin this case, no one in particular.

foundationbriefs.com

Page 15 of 104

Sept/Oct 2013 Topic Analysis One The limits and uses of specifics and hypotheticals. Arguing any resolution involves a good degree of understandingwhat are the implications and examples of its components? For instance, this resolution has two main components: democracy and compulsory voting. The issue of what a democracy truly isand essentially, what a sovereign government isconstitutes a question for which each individual will have a different answer and justification. Compulsory voting, however, is a more concrete idea. The requirement to vote is the basic tenet of any such policy, but sanctions (the penalties coded into law, not the actual enforcement) range from non-existent to fines or jail time, depending on the country. Likewise, enforcement can range from nonexistent to strict. Even further, compulsion may sometimes only cover having to show up to a polling place (and not necessarily having to cast a ballot). For a thorough rundown, Costas Panagopoulos The Calculus of Voting in Compulsory Voting Systems has a rundown of compulsory voting systems in democracies across the world on its fourth page. When crafting a line of argumentation from a moral standpoint, policy specifics dont command much attention. But they are something to keep in mind. If you have a good line of reasoning worked out, taking a look at different compulsory voting systems is a good way of checking your argumentare there instances of compulsory voting systems that you cant account for? Along those same lines, be wary of having an internationally limited approach. It can be easy to assume a U.S.centric paradigmtreating all democracies with the assumption of historically steady voting rates regressing down to around 50% and a two-party system. Many states, including those without compulsory voting systems, do not have that same historical trend or current situation. At the end of the day, handling this resolution will be a fundamentally similar approach to working with any other topic: keep it simple and understand your arguments. With a more straightforward (semantically, at least) topic and a clearly-defined set of stock arguments that people are already familiar with, debates on this topic will likely come down to who can present clear and nuanced arguments.

foundationbriefs.com

Page 16 of 104

Sept/Oct 2013

Topic Analysis Two

Topic Analysis Two


This month will see debaters tackle a topic largely centered in the moral, philosophical world, but also a topic that is very applicable and relevant to the real world. The resolution does not ground the debate in a particular democracy (i.e. the United States) and uses the crucial word ought to place the debate in a hypothetical question of morality. At the same time, democracies around the world are suffering from low voter turnout and questions about how much authority elected governments can claim as a result. Additionally, one can wonder just how effectively these governments are serving their citizenry if so many of those citizens do not fully participate in the democratic process.

On the affirmative side of the debate, the central focus will be on the duties of a citizen in a democracy and whether or not such duties can be justified. Another aspect of the debate will be if requiring voting brings outcomes to a democracy that legitimately outweigh the infraction on liberty (if liberty is impinged at all). When a person agrees to a social contract, they give up absolute liberty to benefit from a society in terms of security. Part of being in the social contract means accepting duties as a citizen of that state: jury duty, military service or the draft, taxes, obeying traffic laws, etc. In each of these cases, the citizen is already being asked to take action that otherwise limits their freedom to act. Certainly, the negative would be ambitious to take on a radical view of libertarianism to the extent that would argue against all of these duties. Indeed, to say that no duties should exist would be to question the social contract and thus democracy under the context of this debate.

Therefore, the affirmatives job is to convince the judge why voting should be added to the list of essential duties in a democracy. The classic conception of democracy sees full participation as crucial to legitimacy of the government and as a necessary act for the full fulfillment of the citizens. In the real world, voluntary voting often leads to the massive underrepresentation of certain groupsparticularly the socioeconomically disadvantaged. Without this representation in the polls, it becomes less likely that policies and government spending help these citizens. Instead, the government can become a servant of the elite. This greatly undermines the reality of a democracy. Another angle of debate is the issue of free loading. The idea here is that people should only experience the benefits of democracy if they work to create and maintain itvoting being central to this maintenance. The key will be to create a compelling and consistent narrative to defend voting as a duty. Again, play up the many duties we already accept and require for a functioning society. One final note: establish

foundationbriefs.com

Page 17 of 104

Sept/Oct 2013 Topic Analysis Two and defend your definition of compulsory voting. It will be highly advantageous to highlight that current conceptions of compulsory voting allow citizens to not cast a vote; they only need to show up at the polls.

The negative will find friends in those who value liberty. The negative should argue that compulsory voting is harmful state paternalismtelling citizens what is best for them. Citizens should be allowed to decide for themselves whether or not they will participate in democracy in that specific way. In fact, citizens may feel that not voting is an important element of democratic participation. Being forced to vote in an election may legitimize a government or electoral system with which a citizen believes to be corrupt or immoral. Abstention is a vital act. The modern conception of liberty is usually that, in a just society, people should be left to act as they wish in private. Forcing people to the polls is a violation of this form of liberty.

While the affirmative may try to argue that compulsory voting would create a more representative democracy, by bringing the socio-economically disadvantagedthose who historically have low participationto the polls, the system created might just reinforce the problem. In the United States, without improving voter registration or the ease of voting, these citizens might still have a hard time voting and now just get punished for it. That would further divide society.

Good luck.

foundationbriefs.com

Page 18 of 104

Aff Evidence

foundationbriefs.com

Page 19 of 104

Sept/Oct 2013

Aff: Increases Turnout

Compulsory voting increases turnout


Voter turnout is higher in Australia than in voluntary voting democracies. RMF Allen, James. "In Praise of Compulsory Voting." Quadrant, May 2012. Web.
At the last federal election in Canada, the country most similar to Australia in terms of its constitutional history, and my native land, the voter turnout was 61 per cent. That was in the middle of last year. The election before that it was 59 per cent. You have to go all the way back to the 1993 election to hit a turnout of close to 70 per cent. Meantime provincial elections in Canada achieve voter turnout rates like 49 per cent (at the Ontario election in October last year) and 40 per cent (at the last Alberta election in 2008) to take what are arguably Canadas two most economically important provinces. And then there is the United Kingdom. The last general election there in 2010 saw voter turnout hit 65 per cent. The one before that was 61 per cent. You have to go back to Winston Churchills return to office in 1951 to see a high figure like 82 per cent. What of the United States? In the last two presidential elections of 2004 and 2008 the turnout rate was 57 per cent (if you look at the voting age population) or 63 per cent (if you look at registered voters); it was 51 per cent (of the voting age population) in 2000; and it was 49 per cent (ditto) back in 1996 when Bill Clinton and his cigars were re-elected to a second term. And if that doesnt seem like a great rate of participation, it looks magnificent compared to voting turnout rates for Congress in non-presidential years. The most recent such rate for the 2010 Congressional elections was 38 per cent; it was 37 per cent for 2006; and 37 per cent for 2002. And against all that comparative data, recall that with our compulsory voting in Australia, federal election turnout rates are always somewhere in the mid-90s. It was 95 per cent at the last election. So even conceding that there are different ways to measure voter turnout rates, that there is room to concede that the figures may not be perfect, and also allowing for any other quibbles anyone may wish to make, what is not arguable is that in Australia a far higher percentage of people of voting age vote to choose their government than in the countries with which we would normally compare ourselves. We are talking about somewhere between an extra fifth to two-fifths of the voting population who have a say in choosing who will govern them here in Australia. Thats an awful lot of people to ignore in the name of some such principle as liberty that proponents of voluntary voting invoke.

foundationbriefs.com

Page 20 of 104

Sept/Oct 2013

Aff: Increases Turnout

Success of Mandatory Voting AMS Armin Shafer. Republican Liberty and Compulsory Voting. Max Planck Institute for the Study of Societies November 2011.
In many established democracies, turnout is declining albeit to different degrees. If we look at 20 countries that have continuously held democratic elections since World War II, we see that turnout in parliamentary elections has declined. () Whereas turnout today is comparatively low in Switzerland or the United States, it still surpasses 80 percent in Denmark, Iceland, and Sweden. In countries with mandatory voting, turnout is even higher. More than 90 percent of eligible voters regularly show up at the polls in Australia, Belgium, and Luxembourg.

foundationbriefs.com

Page 21 of 104

Sept/Oct 2013

Aff: Emphasis on Democratic Procedure

Places emphasis on democratic procedure


Emphasizing democratic procedure allows the votes to be counted, rather than promoting an elite aristocracy that decides what is democratic. RMF Allen, James. "In Praise of Compulsory Voting." Quadrant, May 2012. Web.
When it comes to democracy, broadly speaking there are the thins and the fats. The thins see democracy as a procedural tool for allowing the majority to rule. What you do is count all of us voters as equal and then let the numbers count. Democracy is all about how decisions are taken. It may correlate with very good substantive outcomesthat as an empirical generalisation democracies are more desirable places to live than other places on average, over time (which I believe is true)but there is no necessary connection. Democracies, on the thin understanding, can produce bad substantive outcomes, can even pass illiberal laws. Meanwhile the fats have a different conception of democracy. They take the core idea related to how decisions ought to be made and they stuff it full of moral abstractions; they make it more morally pregnant. So democracy now means not just how decisions are taken. It also includes a judgment related to what those decisions are and whether they are acceptable ones (to them or to some group that is by definition smaller than the voting population as a whole). So the fats now get to assess how rights-respecting some statute passed by the elected legislature was, or whether it was unduly illiberal. And if was too illiberal, well on this new understanding of democracy it just doesnt count as democratic, even though it is a product of the majoritys legislature. I am a thin when it comes to how best to understand democracy. I think its best to understand it as a procedural tool for making decisions. The fat conception is less attractive, to my way of understanding, because it leaves wholly out of sight two things. First off, people disagree about what is and isnt rightsrespecting or liberal or in keeping with liberty or all such substantive calls. Smart, nice, well-informed people simply disagree. And second, if it is not to be all of us counted equally as voters who will decide these substantive moral calls, then at least sometimes it will be the top judges and the overseas committee members of United Nations agencies (and other internationalists) who will now get to make some of the authoritative calls otherwise made by the voters and hence who will, on this fat understanding of democracy, have more say on a host of debatable social policy issues than you and your fellow citizens. So for me its better to be able to say, This is a democracy (given the procedural way in which its government was chosen) but its a bad democracy rather than having to say This government did such bad things that it doesnt get to count as a democracy, which is where the fat understanding inevitably takes you. For thins you

foundationbriefs.com

Page 22 of 104

Sept/Oct 2013 Aff: Emphasis on Democratic Procedure can still debate whether some place is or is not a democracy, but the debate all takes place on the plane of whether the numbers really did count (secret ballots? a chance for all sides to put their case? scrutineers? and so on). Now we can go back to the issue of compulsory voting. If youre a thin about democracy, like me, then what you want is a procedural method for letting the numbers count, for measuring the values, judgments, sentiments and views of the voters and for picking a government that is most in accord with those views.

Compulsory voting upholds the public good of voting in a democracy, DAT Lever, Annabelle. Compulsory Voting: A Critical Perspective. British Journal of Political Science (2009).
The key idea here is that a democratic electoral system is a public good, in that all citizens get to benefit from it, even if they do nothing to contribute to it. Because it is a public good, it is possible to free-ride, or to enjoy the benefits of that good, without contributing oneself and, indeed, most people will have an interest in doing precisely that. Non-voters, therefore, can be seen as free-riders, selfishly and immorally exploiting voters. The moral force of this point is two-fold. First, it reinforces the idea that no morally significant liberties are threatened by compulsory turnout and, secondly, it carries the battle into the enemy camp. It is selfish and exploitative to benefit from the efforts of other people without making any effort to contribute. So, far from compulsion being unjustified, or even morally neutral, it seems positively desirable, as a curb on selfish and exploitative behaviour. As Lijphart puts it, It must be remembered that nonvoting is a form of free riding and that free riding of any kind may be rational but is also selfish and immoral. The normative objection to compulsory voting has an immediate intuitive appeal that is not persuasive when considered more carefully. (1)

foundationbriefs.com

Page 23 of 104

Sept/Oct 2013

Aff: Emphasis on Democratic Procedure

CV upholds democratic ideals, e.g. more clean and equitable campaigning, DAT The Case for Compulsory Voting in the United States. Harvard Law Review, Vol. 121:591. 2007. Web. http://harvardlawreview.org/media/pdf/compulsory_voting.pdf
In addition to the direct effect of compulsory voting on turnout, there are also several indirect benefits. First, compulsory voting would reduce the role of money in politics. Political parties would not spend as much money on their get-out-the-vote efforts since high turnout would already be ensured and would be fairly inelastic.36 Some of the get-out-the-vote money could be shifted to other forms of campaign spending, but not all of it. A significant amount of spending on getting out the vote comes from groups known as 527s (a reference to the tax code) and nonpartisan groups that are not subject to campaign finance laws. These groups are limited in their abilities to campaign expressly in favor of candidates. Presumably, these organizations would shift some funds from getting out the vote to issue ads (which are permissible), but the diminishing marginal effectiveness of those ads would limit this. With this implicit limit on spending, politicians and parties might focus somewhat less on fundraising and be less beholden to donors. Another indirect benefit of compulsory voting is that it might lead to the kinds of changes in American political culture that could increase political awareness and engagement. A compulsory voting regime would change the ways in which candidates, political parties, and other political groups develop campaign strategies. For example, compulsory voting might lead to fewer negative campaigns featuring attack ads because such ads generally succeed by selectively lowering turnout among targeted groups. Once the prospect of significantly lower voter turnout is removed, candidates would presumably reduce or eliminate the use of this tactic and focus on different, perhaps qualitatively superior, tactics. More generally, the current political discourse has developed in a system in which relatively few people vote and those who do have relatively homogeneous demographic characteristics. Political organizations have developed campaign messages and strategies that are successful at appealing to those voters. Compulsory voting would bring a new population into play, and would force political actors to make changes in their campaign methods in order to take these new voters into account whether those changes involve their substantive policy positions or the means of communicating those positions. (597)

foundationbriefs.com

Page 24 of 104

Sept/Oct 2013

Aff: Emphasis on Democratic Procedure

CV encourages voting from marginalized groups in society JMR Chong, Alberto and Mauricio Olivera. On Compulsory Voting and Income Inequality in a Cross Section of Countires. Inter-American Development Bank. May 2005. Pg 6 .
Nonetheless, those who would benefit the most by voting tend not to vote, as their transaction costs tend to be extremely high. Furthermore, the marginal impact of a single voters choices, especially in non-urban areas, is rightly or wrongly perceived as inconsequential. This biases individuals cost-benefit calculation against voting, as individuals do not adequately assess the positive externalities of voting outcomes, which are typically not internalized in the utility functions of the voter.2 In this context, compulsory voting mechanisms have been encouraged to make voting turnout more representative of the democratic process.

CV Ensures Better and more Accurate Representation JMR Chong, Alberto and Mauricio Olivera. On Compulsory Voting and Income Inequality in a Cross Section of Countires. Inter-American Development Bank. May 2005. Pg 8-9.
Likewise, some partial evidence sustains the connection between turnout and representation. As differentials in turnout based on socioeconomic characteristics diminish, support for leftist parties increases (Pacek and Radcliff, 1995; Nagel, 1998). It has been shown that higher turnout benefits leftist parties, while lower turnout appears to favor parties of the right (McAllister, 1986). Furthermore, income is consistently, positively correlated with the probability of voting, while education has proven to be positively and significantly related to voter turnout in virtually every study of voter participation (Muller, 1989).5 Also, it has been argued that compulsory voting countries and non-compulsory voting countries have the same mean voter turnout, but differ in the prevalence of democratic institutions; the compulsory- voting group has a greater democratic presence than the voluntary voting group. A sample of countries with a greater prevalence of democratic institutions exhibits greater sensitivity to voter preferences than a sample of basically autocratic countries. That is, insofar as government consumption consists of public goods and redistributive transfers, democracies are more sensitive to the general needs of the populace than autocratic regimes, where greater shares of increasing gross domestic product may be transferred to the private wealth of the ruling elite (Yeret, 1995).

foundationbriefs.com

Page 25 of 104

Sept/Oct 2013

Aff: Enhances Election Process

Enhances election process


Guaranteed turnout means candidates focus on right issues during campaign. RMF Lever, Annabelle. 2009. Is Compulsory Voting Justified?. Public Reason 1 (1): 57-74.
It may cut down the cost of campaigns, encourage politicians to engage with those who are least interested in politics, and it may minimize negative campaigning, as well. The idea behind these potentially attractive features of compulsion is that if everyone has to vote, politicians can largely take turnout for granted, but have an especial interest in ensuring that those who turn out do not vote for the other side. In short, compulsion means that the battle is not, any more, to make sure that your supporters actually get to the polls, or to deter those of your opponents from doing so, (apparently the chief effect of negative campaigns), but to ensure that of those who turn out, as many vote for you as possible (61)

CV encourages candidates to engage with those Alienated in the Political System JMR Keaney, Emily and Ben Rogers. A Citizens Duty Voter Inequality and the Case for Compulsory Turnout. Institute for Public Policy Research. May 2006. Pg 29
Compulsory turnout not only increases turnout, it also cuts down the cost of political campaigning and encourages the political parties to engage with those groups least interested in politics or most dissatisfied with the political system. Where turnout is voluntary, most political parties focus on motivating their supporters to vote, rather than winning the support of undecided voters. Both national and local campaigning tend to be directed to this end. Where turnout is compulsory, however, parties can generally rely on their supporters turning out. This can reduce the cost of electioneering and/or encourage parties to concentrate on winning over people who do not support any political party people who often feel alienated from the political system. This in turn can increase the publics sense of political efficacy and their confidence in the political system (Lijphart 1997).

foundationbriefs.com

Page 26 of 104

Sept/Oct 2013

Aff: Turnout Inequality Harms Democracy

Turnout inequality harms democracy


Low turnout harms legitimacy of government. RMF Lever, Annabelle. 2009. Is Compulsory Voting Justified?. Public Reason 1 (1): 57-74.
Lower turnout seems to threaten the legitimacy of a countrys government and electoral system, because it significantly increases the likelihood that governments will reflect a minority, rather than a majority, of registered voters, and of the voting-population, itself. As Ferdinand Mount said, commenting on the report of the Power Inquiry, in Britain, when little more than 20% of the electorate has voted for the winning party, as in the United Kingdom general election of May 2005, legitimacy begins to drain away. He adds, If only just over half of us bother to vote at all in national elections and scarcely a third in local elections, the bureaucracy begins to think of elections as a tiresome and increasingly insignificant interruption in its continuous exercise of power. What develops isexecutive democracy and . . . more rudely described . . . elective dictatorship (Mount 2006). (58-59)

Voluntary turnout is skewed toward the elite. RMF Lever, Annabelle. 2009. Is Compulsory Voting Justified?. Public Reason 1 (1): 57-74.
It is not news that turnout has been declining in most democracies since the Second World War. However, the association of low turnout with unequal turnout may be less well known and its significance less clearly appreciated. For example, in the last two General Elections in Britain the participation gap between manual and non-manual workers more than doubled: from around 5% in 1997 to around 11% in 2005. Likewise, between the 1960s and 2005 the difference in turnout between the top and bottom quartile of earners grew from 7% to around 13%. The results are not dissimilar in other countries, and are particularly pronounced in the United States, where turnout at presidential elections for the college educated can be over 25% higher than that of the population as a whole, while those who lack a high-school diploma are 16% less likely to vote than the general population (Rose 2000, 316-7). (59)

foundationbriefs.com

Page 27 of 104

Sept/Oct 2013

Aff: Turnout Inequality Harms Democracy

Low turnout harms the socioeconomically disadvantaged. RMF Lever, Annabelle. 2009. Is Compulsory Voting Justified?. Public Reason 1 (1): 57-74.
So, it looks as though those people who do least well in our societies are least likely to vote; and in what seems to be a vicious circle, those least likely to vote are least likely to attract sympathetic attention from politicians eager to get elected or reelected. So inequalities in turnout are troubling, because they suggest a vicious circle in which the most marginal members of society are further marginalized . Not only that: in so far as these non-voters are more likely to vote for social democratic polities than other people, and particularly likely to benefit from them, inequalities in turnout seem to deprive the left of a significant political constituency and make it easier for the right to get reelected. Hence, as Lijphart makes plain, social democrats should be particularly concerned about declining voter turnout because it makes it more difficult to elect social democratic governments and, therefore, to pass social democratic legislation or public policies. (5960)

Political participation places positive pressures on democratic government, and vice versa DAT Verba, Sidney. Thoughts About Political Equality. Harvard Kennedy School of Government, 2001. Web.
The two parties are now trying to bring out the vote -- in order to elect themselves and reap the selective benefits of office. Thats part of it. But they are doing do to pursue an agenda that they believe in the interest of the people they are mobilizing. They need to mobilize them on the basis of things that appeal to them. We care about equality of political influence because it is important. Societies with healthy democracies in which there is real control by citizens over the government -- as there is in the U.S. though it is hard to say how much are better societies. They do not commit massive crimes against their own citizens -- or are at least less likely to and they will be less massive. Amartya Sen has made the major and very striking point that famines do not take place in democracies -- since famines come less from a sheer absence of food than from an inadequate distributional mechanism. (Sen, 2000). And democracies are under some pressure to provide such mechanisms. Political participation -- open and free -- makes the state of the society clear; thats one step to the government being responsive. And it makes clear that there are negative consequences for the government if it does not respond. Thats the second step to responsiveness.

foundationbriefs.com

Page 28 of 104

Sept/Oct 2013

Aff: Turnout Inequality Harms Democracy

Benefits of Compulsory Voting Outweigh Disadvantages AMS Institute for Public Policy Research. Young Voters Should be Required to Vote First Time Around. August 26, 2013. http://www.ippr.org/pressreleases/111/11175/young-voters-should-be-required-to-vote-first-time-round
Sarah Birch, Professor of Politics at the University of Glasgow, and co-author of the report said: Unless radical action is taken, turnout inequalities in Britain will grow wider. First time compulsory voting could well be very effective in engaging young people in politics. There are many other things that young people are required to do, not the least of which is go to school. Adding just one more small task to this list would not represent an undue burden, and it could well help to reinvigorate democracy. It would make politicians target first-time voters like never before and give young voters the potential for far greater political power.

Unequal Representation AMS Institute for Public Policy Research. Young Voters Should be Required to Vote First Time Around. August 26, 2013. http://www.ippr.org/pressreleases/111/11175/young-voters-should-be-required-to-vote-first-time-round
Guy Lodge, IPPR Associate Director, said: Unequal turnout matters because it gives older and more affluent voters disproportionate influence at the ballot box. Turnout rates among the young have fallen significantly which means there is less incentive for politicians to pay attention to them. Young people who dont vote today are less likely than previous generations to develop the habit of voting as they get older, which is why first time compulsory voting is so important. Unequal turnout unleashes a vicious cycle of disaffection and under-representation. As policy becomes less responsive to their interests, more and more decide that politics has little to say to them.

foundationbriefs.com

Page 29 of 104

Sept/Oct 2013

Aff: Turnout Inequality Harms Democracy

Inequality Problems in UK AMS Institute for Public Policy Research. Young Voters Should be Required to Vote First Time Around. August 26, 2013. http://www.ippr.org/pressreleases/111/11175/young-voters-should-be-required-to-vote-first-time-round
IPPRs report shows that only 44 per cent of 18-24 year olds voted in the 2010 General Election, compared to 76 per cent of those aged 65 and over. Turnout inequality between young and old voters has grown at an alarming rate in recent years and shows little sign of being reversed: the 18 point turnout gap between 18-24 year olds and those aged over 65 that existed in 1970 had grown to a 32 point gap by 2010. IPPRs report says voting matters as there are fewer incentives for the Government to respond to the interests of non-voting groups. The spending cuts have disproportionately affected young people, with 1624 year olds facing cuts to services worth 28% of their annual household income, compared to 10% of the income of those aged 55-74.IPPRs analysis shows that overall those who did not vote in the 2010 general election faced cuts worth 20% of their annual household income, compared to 12% of those who did vote. Those with annual household income under 10,000 stand to lose 41 per cent of their average income. By contrast those with income over 60,000 will lose on average 2,104, which represents only 3 per cent of this groups income.

Problems with Unequal Participation AMS Alex White. June 14, 2013. New Matilda. https://newmatilda.com/2013/01/14/defencecompulsory-voting
Unequal voter participation has other negative consequences. The people least likely to vote under voluntary systems are people from the least privileged backgrounds. They are people with low incomes, or from ethnic backgrounds, or with less education. In most industrialised countries, it is age and education that have most impact on whether you vote or not. This means that younger people are less likely to vote not because they're lazy, but because they have less access to free time, less income, less time to devote to civic activities (not all young people are Arts students after all).

foundationbriefs.com

Page 30 of 104

Sept/Oct 2013

Aff: Turnout Inequality Harms Democracy

Unequal turnout reinforces gaps in privilege, DAT 9/9 11:39pm Lever, Annabelle. Compulsory Voting: A Critical Perspective. British Journal of Political Science (2009).
It is not news that turnout has been declining in most democracies since the Second World War. However, the association of low turnout with unequal turnout may be less well known, and its significance less clearly appreciated. For example, in the last two General Elections in Britain the participation gap between manual and non-manual workers more than doubled: from around 5% in 1997 to around 11% in 2005. Likewise, between the 1960s and 2005 the difference in turnout between the top and bottom quartile of earners grew from 7% to around 13%. These results are not dissimilar in other countries. The fact that lower turnout means increasingly unequal turnout is troubling, because those least likely to turn out are overwhelmingly drawn from the least privileged social groups in a polity. Thus, the IPPR report notes that though socio-economic status - whether measured by income, class or education is not as significant a factor as age in determining whether a person will vote or not, it has nevertheless become an increasingly significant factor at least in the UK. .although there has been some decline in turnout among all income categories since 1964, the decline is most rapid for those with the lowest income. So, it looks as though those people who do least well in our societies are least likely to vote; and in what seems to be a vicious circle, those least likely to vote are least likely to attract sympathetic attention from politicians eager to get elected or reelected. Inequalities in turnout are troubling, then, because they suggest a vicious circle in which the most marginal members of society are further marginalized. (1)

foundationbriefs.com

Page 31 of 104

Sept/Oct 2013

Aff: Turnout Inequality Harms Democracy

Mandatory voting is rooted in the concept of universal suffrage, DAT 9/10 12:08am Malkopoulou, Anthoula. Lost Voters: Participation in EU Elections and the Case for Compulsory Voting. Center for European Policy Studies (July 2009).
Compulsory voting was introduced for the first time on a national scale in Belgium in 1893. A rather technical reason at that time was to protect poor voters from forced abstention, i.e. negative vote-buying, by their employers. Although this is not an issue in most European countries today, it may still be a useful mechanism to prevent electoral corruption and abstention-buying in countries that feature large economic divides and labour dependence. The main rationale behind the system in Belgium was to complement and enforce universal suffrage, which was introduced in the same constitutional reform of 1893. Compulsory voting was a way to fulfil the principle of political integration, in other words it was a method to politically unite a socially disparate people. In this sense, if the principle of universality is a central aspect of voting rights, its progressive realisation requires respect from the state and protection from third-party interference. As with all human rights, the third and most advanced step is fulfilling such rights, in other words providing guarantees that they will be exercised. So, full political integration is perhaps the strongest argument for compulsory voting. Indeed, the most important implication of universal participation is political equality. In his milestone article in 1997, Arend Lijphart argued that low turnout is biased against citizens with a lower education, income and social class. According to him, citizens with lower education or modest social status, as well as those belonging to ethnic, linguistic or religious minorities, are more prone to abstention than others. Conversely, voluntary voting perpetuates political inequalities and misrepresentation. Paradoxically, the claim of abstainers that the European Parliament is an elitist establishment is reinforced by abstention itself. (1)

Law of Dispersion AMS Armin Shafer. Republican Liberty and Compulsory Voting. Max Planck Institute for the Study of Societies November 2011.
The main reason for a concern with falling turnout is that lower participation rates mean more unequal participation, since voters and non-voters are not evenly distributed in society. People with lower incomes or education generally have a lower propensity to vote than the better-off and with a lower average turnout this gap widens (Lijphart 1997). This empirical regularity prompted Tingsten ([1937]1975: 230) to formulate the law of dispersion, which postulates that the differences in electoral participation among social groups are smaller if the overall participation rate is higher.

foundationbriefs.com

Page 32 of 104

Sept/Oct 2013

Aff: Turnout Inequality Harms Democracy

Compulsory voting brings the underrepresented to the voting booth. RMF Selb, Peter, and Romain Lachat. "The More, the Better? Counterfactual Evidence on the Effect of Compulsory Voting on the Consistency of Party Choice." European Journal of Political Research (2009). Web.
Much research has shown that higher levels of education, political interest and sophistication increase the probability of turning out to vote (Wolfinger & Rosenstone 1980; Powell 1986; Brady 1995; Jackson 1995).Thus, in a voluntary voting system, citizens who participate should come disproportionately from those strata with high education, interest and sophistication. Under CV, the bias should be weaker. This has been illustrated by Jackman (2003), for example, who compares Australia and the United States. In the latter, citizens with a low level of political sophistication are strongly underrepresented among voters. In Australia, by contrast, CV brings more citizens with a low level of political sophistication to the polls (575)

Compulsory voting can eliminate voting inequalities, DAT Hill, Lisa. Compulsory Voting in Australia: A Basis for a Best Practice Regime. Federal Law Review, vol. 32.
Aside from its ability to significantly improve turnout, the second major benefit of compulsory voting is its capacity to close, more or less completely, the SES voting gap that exists in most voluntary systems. The effectiveness of compulsory voting at closing this gap has been demonstrated empirically. In Belgium socioeconomic status, which is normally a determinant of non-voting, is 'effectively erased' as a variable for non-voting under a system of compulsion. Similar results were found in studies conducted in The Netherlands, a country that provides particularly compelling evidence of this closure due to the abandonment of compulsory voting in the seventies. In Australia, a number of studies addressing the question of how turnout would look under a voluntary regime conclude that a class bias would most likely benefit the more conservative Liberal-National Coalition. (486)

Trend of Increasing Voter Inequality JMR Rogers, Ben. Turnout is really about class. The Guardian, May 2005.
That the rich vote more than the poor is a common-place of political science. The difference is notoriously pronounced in the US, but it is pretty universal - in most European countries the working class turnout trails upper class turnout by about 10%. Yet that hardly makes it less significant. According to Mori, 70% of voters from the top social classes voted last week, but only 54% of from the bottom did. Moreover, the gap appears to be growing, increasing from 13% in 1997 to 15% in 2001 and 16% last week. The political voice of the well off remains strong, as that of the poor gets weaker.

foundationbriefs.com

Page 33 of 104

Sept/Oct 2013

Aff: Turnout Inequality Harms Democracy

Democracies must remain majoritarian, DAT Engelen, Bart. Why Compulsory Voting Can Enhance Democracy. Acta Politica, 2007, 42.
Political participation is also crucial for guaranteeing the legitimacy of a democratic regime. The more citizens abstain, the more the elected bodies lose their accountability. To illustrate the problem one can refer to elections where only a minority of the electorate determines the electoral result. In elections to the European Parliament, for example, average turnout has declined systematically from 63% of all registered voters in 1979 to a record low of 45.6% in 2004 (EP, 2004). As more than half of the electorate abstains in 18 of the 25 member states, one can hardly speak of popular or majority will (Watson and Tami, 2001). As democracy cannot imply that laws are enacted by legislators representing a minority of eligible voters, one has to conclude that high turnout levels are necessary for any democracy claiming legitimacy.

High Voter Turnout Creates Stability JMR Lijphart, Arend. Unequal Participation: Democracys Unresolved Dilemma. The American Political Science review, Volume 91, Issue 1. March 1997. Pg 10.
Tingsten (1937) already used the Weimar example to warn that exceptionally high voting frequency may indicate an intensification of political conflict that may foreshadow the fall of democracy. The danger is that, in periods of crisis, sudden jumps in turnout mean that many previously uninterested and uninvolved citizens will come to the polls and will support extremist parties. This however, is an argument for, not against compulsory voting: Instead of trying to keep turnout at steady low levels, it is better to safeguard against the danger of sudden sharp increases by maintaining steady high levels, unaffected by crisis and charismatic leaders. Additional evidence that the Weimar precedent should not discourage efforts to increase turnout is Powells (1982, 206) comparative study of 29 democracies in which he found a strong association between higher voter turnout and less citizen turmoil and violence: The data favor the theorists who believe that citizens involvement enhances legitimacy instead of producing democratic breakdown.

foundationbriefs.com

Page 34 of 104

Sept/Oct 2013

Aff: CV Decreases Income Inequality

CV decreases income inequality


Summary of study: Repealing compulsory voting in Venezuela increased income inequality. RMF Carey, John, and Yusaku Horiuchi. Compulsory Voting and Income Inequality. Rep. Dartmouth College, 22 Apr. 2013. Web.
What difference does it make if more, or fewer, people vote? What difference would it make if the state makes people vote? These questions are central both to normative debates about the rights and duties of citizens in a democracy and to contemporary policy debates in a variety of countries over what actions states should take to encourage electoral participation. To address them, this paper focuses on the phenomenon of compulsory voting legal requirements that compel citizens to vote in elections. Specifically, by applying a new statistical tool called synthetic control method to a rare case of abolishing compulsory voting in Venezuela, we show that not forcing people to vote and a resultant sharp drop in voter turnout yielded a more unequal distribution of income. (2)

Theoretical background on why this happens: differences in voting costs/benefits. RMF Carey, John, and Yusaku Horiuchi. Compulsory Voting and Income Inequality. Rep. Dartmouth College, 22 Apr. 2013. Web.
The first assumption is that when voting is voluntary, wealthier people vote at higher rates than do poorer ones. This pattern has been found to be empirically robust over time and across most countries (Jackson, Brown, and Wright 1998; Leighley and Nagler 1992; Singh 2011; Tingsten 1937)...The potential benefits of voting are assumed to be constant across citizens,2 but the costs are higher and thus, overall utility is lower for the poor. (5)

foundationbriefs.com

Page 35 of 104

Sept/Oct 2013

Aff: CV Decreases Income Inequality

Circumstances of the study. RMF Carey, John, and Yusaku Horiuchi. Compulsory Voting and Income Inequality. Rep. Dartmouth College, 22 Apr. 2013. Web.
With regard to the timing of intervention, we need to fill in the details. Compulsory voting was enshrined in Venezuela in the Constitution of 1961 and remained constitutionally required until the adoption of the new Constitution of 1999. In the mid-1980s, public disillusionment with Venezuelas dominant political parties increased, with broad sentiment that the parties were unable to formulate effective economic policies and perceptions of widespread corruption. A manifestation of this disillusionment was an increase in demands that citizens ought to have the right to choose whether to vote or not. During this period, the electoral authorities scaled back enforcement of the sanctions associated with compulsory voting, and finally, the Organic Suffrage Law of 1993 eliminated the sanctions (Molina and Perez Baralt 1995, 1996). In the Constitution, the duty to vote remained until the promulgation of the new charter in 1999, but the requirement was enforced more aggressively during the 1960s and 1970s than subsequently, and backed by no legal sanctions from 1993 on.12 After the abolishment of enforced compulsory voting in Venezuela in 1993, voter turnout sharply dropped. In Parliamentary elections in Venezuela, voter turnout had been consistently over 70% previously, but dropped to 50.0% in 1993. 13 For theses reasons, we focus on 1993 as the critical year when a major policy shift occurred. (13-14)

Removal of CV increased income inequality in Venezuela. RMF Carey, John, and Yusaku Horiuchi. Compulsory Voting and Income Inequality. Rep. Dartmouth College, 22 Apr. 2013. Web.
This is equivalent to say that the effect of the 1993 intervention in Venezuela on income equality is significant at 10% level.Given the results of these two placebo tests, we are inclined to believe that the intervention in 1993 in Venezuela caused an unusually sharp increase in income inequality during the post-intervention period. (18) In sum, all these results suggest that the removal of legal sanctions for non-voting in Venezuela in 1993 fueled the observed increase in income equality after 1993. (19) The authors then go on to examine the political and economic landscape of Venezuela at the time and find that, at least for the period surrounding the repeal of compulsory voting, it seems unlikely that other trends would completely nullify the effect they observed. Also, significant at the 10% level essentially means that there is a less than ten percent chance that the observed effect (in this case increased income inequality) is due to something other than the studys variable (repeal of compulsory voting).

foundationbriefs.com

Page 36 of 104

Sept/Oct 2013

Aff: CV Decreases Income Inequality

CV Influences Policy so that it Decreases Income Inequality JMR Armin Shafer. Republican Liberty and Compulsory Voting. Max Planck Institute for the Study of Societies. November 2011. Pg 13.
Cross-national studies also show that turnout influences policy outcomes. Hicks and Swank (1992) report that higher turnout leads to higher welfare spending, and Mahler (2008) demonstrates that high-turnout countries redistribute more. Similarly, Mueller and Stratmann (2003) present evidence for a large set of countries that higher rates of electoral participation tend to equalize incomes while at the same time decelerating growth. Looking more specifically at the effects of compulsory voting, Chong and Olivera (2008) are able to show that countries that legally oblige citizens to vote distribute incomes less unequally. In contrast to empirical studies that look at the opinions of voters and non-voters, these studies suggest that the level of turnout matters and the introduction of compulsory voting would not remain inconsequential. Voters and non- voters might diverge only slightly in their preferences, but if parties consistently seek to address the median voter rather than the median citizen, these differences could accumulate over time and translate into different policy outcomes.

Eliminating CV Increased Voter Inequality JMR Rogers, Ben. Turnout is really about class. The Guardian, May 2005.
In addition to Australia and many South American countries, there are six European countries with compulsory voting. It tends to increase participation at the ballot box by 10-15% and narrow the gap between the rates at which rich and poor vote. Before the Netherlands abolished compulsory voting in 1970 there was only a 4% difference between the voting levels of the top and bottom classes. After abolition the difference jumped to 21%.

foundationbriefs.com

Page 37 of 104

Sept/Oct 2013

Aff: Success Examples

Success Examples
Success in Belgium AMS Institute for Public Policy Research. Young Voters Should be Required to Vote First Time Around. August 26, 2013. http://www.ippr.org/pressreleases/111/11175/young-voters-should-be-required-to-vote-first-time-round
The report shows that people who vote in the first election they are eligible for are more likely to continue voting throughout their life. The report shows that countries that have compulsory voting are successful at maintaining high levels of participation. In Belgium where compulsory voting is law but not enforced the voter turnout for those under the age of 20 is 87%. The difference in voter turnout between old and young and rich and poor voters is almost non-existent.

Queensland, Australia Success AMS Alex White. June 14, 2013. New Matilda. https://newmatilda.com/2013/01/14/defencecompulsory-voting
Queensland was the first state in Australia to introduce compulsory voting. It did so in 1915 and nine years later, Western Australian Nationalist Party MP Edward Mann introduced the bill to expand compulsory voting across Australia. (...) In this country, unlike in America, we do not worship our constitution. Whereas the US constitution proclaims a government by "We the people", in Australia, we more prosaically note that "The House of Representatives shall be composed of members directly chosen by the people of the Commonwealth". It's worth emphasising that it says "directly chosen by the people". It doesn't say "chosen by half of the people", or "15 per cent of the people". When voting gets as low as 28 per cent, democracy is endangered. This is what the conservative government realised in 1924. Low participation entrenches inequality. It threatens the legitimacy of our country's government because a low vote ensures that only a minority is represented instead of a majority.

foundationbriefs.com

Page 38 of 104

Sept/Oct 2013

Aff: Success Examples

Australias Success AMS Lisa Hill and Jonathon Louth: Compulsory Voting Laws and Turnout. Discipline of Politics, School of History and Politics,University of Adelaide. October 2004. http://www.academia.edu/703084/Compulsory_voting_laws_and_turnout_efficacy_ and_appropriateness
The Australian Experiment. Queensland was the first Australian state to introduce compulsory voting in 1914. Compulsory enrolment for Federal elections was introduced in 1911 but voting itself did not become mandatory until 1924 (A.E.C.,1999). It was not compulsory for Aboriginal people and Torres Strait Islanders to register and vote until 1984.Compulsory voting was introduced in Australia to address the problem of low voter turnout and it proved to be an extremely effective and well-tolerated remedy .In 1903 the federal election turnout had only been 46.86% (AEC 1999) and at the last Federal election immediately prior to the introduction of compulsory voting (1922)the average turnout of registered voters was 58.67%. But turnout at the first federal election after 1924 (ie in 1925) surged dramatically to an average of 91.35%(RV). Data taken from the nine elections preceeding and the nine following itsintroduction shows that the average voter turnout increased by 30.4 percent(Jackman, 2001). Further, turnout rates among the voting age population in Australia have remained consistently high and against the trend of steadily declining voting participation in advanced democracies worldwide (Blais, 2000; Gray and Caul, 2000).In the post-war period (1946-1998) the average turnout rate has been around 83% ofvoting age population (VAP) and 94.51% of registered voters (RV) for the period1946-2001.

foundationbriefs.com

Page 39 of 104

Sept/Oct 2013

Aff: Low Turnout

Low Turnout
Expanding the Vote is Necessary AMS Alex White. June 14, 2013. New Matilda. https://newmatilda.com/2013/01/14/defencecompulsory-voting
Low turnout is a problem for almost every country in the world especially our closest cultural peers, the USA and UK. Even with rockstar candidates like Obama, participation rarely peaks about 65 per cent. Voter turnout fell from 62.3 per cent in 2008 to an estimated 57.5 per cent in 2012. What's more, the 2012 election saw unprecedented attempts by Republicans, bankrolled by conservative billionaires and multinational corporations, to suppress the vote of African Americans, Latinos and young people. Expanding the number of people who vote is not only virtuous in itself. It helps defend our democracy and ensures it is more likely to act in the interests of all the people, not a moneyed minority.

Low turnout undermines the idea of representative government, DAT Engelen, Bart. Why Compulsory Voting Can Enhance Democracy. Acta Politica, 2007, 42.
The more citizens abstain, the less representative the electoral result becomes. This problem is aggravated by the fact that low voter turnout means unequal and socio-economically biased turnout (Lijphart, 1997, 2). As Keaney and Rogers (2006, 10) aptly summarize, international evidence shows that turnout and inequality are closely linked, and that as turnout falls so it becomes more unequal. While universal suffrage equalizes the opportunities for participation in elections (every citizen has exactly one vote), inequalities between privileged and less privileged citizens persist in the way citizens exercise such opportunities (Lijphart, 1998, 12; Verba, Nie and Kim, 1978, 16). This forms a problem because governments normally respond to the opinions expressed by citizens in elections: if you dont vote, you dont count (Burnham, 1987, 99). This assumption has been empirically confirmed. In their cross-country comparison, Mueller and Stratmann (2003, 2151) found that political participation has a positive impact on income equality. The more citizens abstain, the greater income inequality will become. The logic behind this is clear enough. As turnout declines, less privileged citizens tend to abstain more than others. As a result, they will have less representatives looking after their concerns and will therefore exert less influence on policy decisions. As low turnout means unequal influence, it violates the value of political equality, which lies at the heart of any notion of democracy. (2425)

foundationbriefs.com

Page 40 of 104

Sept/Oct 2013

Aff: Encourages Moderacy

Mandatory Voting Encourages Moderacy


More Voting Tempers Politics AMS Eric Liu. Why Voting Should Be Mandatory. Times. August 21, 2012. http://ideas.time.com/2012/08/21/should-voting-be-mandatory/
Second, as William Galston of the Brookings Institution argues, it would temper the polarization of our politics. In todays electorate, hardcore partisan believers are over-represented; independents and moderates are under-represented. If the full range of voters actually voted, our political leaders, who are exquisitely attuned followers, would go where the votes are: away from the extremes. And they would become more responsive to the younger, poorer and less educated Americans who dont currently vote.

Political Parties Encourage Divisiveness by Seeking Greater Turnout AMS Norman Ornstein. Vote--Or else. New York Times. 2006. http://www.nytimes.com/2006/08/10/opinion/10ornstein.html
Heres why. With participation rates of about 10 percent or less of the eligible electorate in many primaries to 35 percent or so in midterm general elections to 50 percent or 60 percent in presidential contests, the name of the game for parties is turnout and the key to success is turning out ones ideological base. Whichever party does a better job getting its base to the polls reaps the rewards of majority status. And whats the best way to get your base to show up at the polls? Focus on divisive issues that underscore the differences between the parties.

foundationbriefs.com

Page 41 of 104

Sept/Oct 2013

Aff: Prevents Free-Riding

Prevents Free-Riding
Democracy is a Public GoodCV Prevents Freeriding JRM Lever, Annabelle. 2009. Is Compulsory Voting Justified? Public Reason 1 (1): 57-74.
The final, and crucial, step in the case for compulsion is the claim that non-voters are free-riding on voters. They are, it is claimed, selfishly benefiting from the public good of a democratic electoral system without doing their part to maintain it. This claim can be found in every argument for compulsory voting, although it is rarely spelled out in any detail. The key idea here is that a democratic electoral system is a public good, in that all citizens get to benefit from it, even if they do nothing to contribute to it. Because it is a public good, it is possible to free-ride, or to enjoy the benefits of that good, without contributing oneself and, indeed, most people will have an interest in doing precisely that. Non-voters, therefore, can be seen as free-riders, selfishly and immorally exploiting voters. The moral force of this point is two-fold. First, it reinforces the idea that no morally significant liberties are threatened by compulsory turnout and, secondly, it carries the battle into the enemy camp. It is selfish and exploitative to benefit from the efforts of other people without making any effort to contribute. So, far from compulsion being unjustified, or even morally neutral, it seems positively desirable, as a curb on selfish and exploitative behaviour. As Lijphart puts it, It must be remembered that nonvoting is a form of free riding and that free riding of any kind may be rational but is also selfish and immoral. The normative objection to compulsory voting has an immediate intuitive appeal that is not persuasive when considered more carefully. (Pg. 10) This card can be used to build a classic argument for government regulation which, in this case, would take the form of compulsory voting. The idea is that public goods must be protected and regulated as the failure to do so can lead to their depletion or ruin. As other cards in the brief have outlined, there are clear costs to the so-called free-riding spelled out above. Thus, the argument is that the public nature of a functioning democracy is not self-sustaining and must be guarded and enhanced. Compulsory voting is an effective way to do so.

foundationbriefs.com

Page 42 of 104

Sept/Oct 2013

Aff: Prevents Free-Riding

CV shifts the position of the median voter JMR Chong, Alberto and Mauricio Olivera. On Compulsory Voting and Income Inequality in a Cross Section of Countires. Inter-American Development Bank. May 2005. Pg 7-8.
In the absence of compulsory voting, those individuals who may benefit the most by voting somewhat paradoxically tend not to vote. This may be in part because related transaction costs tend to be extremely high for this group, as individuals do not adequately assess the positive externalities of voting outcomes. Lack of information or misinformation may be a source of problems as well. In fact, there is ample evidence that the sample of individuals who vote voluntarily is not representative of the entire population of eligible voters. When a compulsory voting rule brings in those voters who would not vote voluntarily, this changes the demographic composition of the effective electorate. Extensive data on income, age, education, race, and so on suggest that these characteristics differ between voters and the remaining eligible population (Crain and Leonard, 1993). In fact, it has been argued that eligible non-voters tend to be poorer, less educated, younger, and disproportionately composed of minority group members when compared to voluntary voters. Particularly in developing countries, chances are that non-voters are net recipients of government services. Compulsory rules may thus increase the share of high-demanders for public services among the active electorate, since the characteristics of voluntary non-voters suggest they are net gainers of government expenditure; bringing these non-voters into the active electorate would shift the position of the median voter.

Voluntary voting is less representative of the median voter JMR Armin Shafer. Republican Liberty and Compulsory Voting. Max Planck Institute for the Study of Societies. November 2011. Pg 13.
What is more, the causal arrow between turnout rates and income inequality seems to run in both directions. Lower turnout leads to more inequality, as just discussed, while rising inequality in turn depresses electoral turnout (Anderson/Beramendi 2008; Solt 2008, 2010). Under these circumstances, a vicious circle might arise, in which decision-makers direct policies at a median voter who less and less resembles the average citizen (Hill 2006: 216). As a result, egalitarian policies higher taxes for the rich, urban development in deprived areas, universal health care, or public investment in early childhood education become less likely, while larger sections of the lower classes might feel alienated from politics and conclude that politics is simply not a game worth playing (Solt 2008: 58), which will make voters even less representative. Under voluntary voting, it is hard to see how this circle can be broken.

foundationbriefs.com

Page 43 of 104

Neg Evidence

foundationbriefs.com

Page 44 of 104

Sept/Oct 2013

Neg: Infringement on Personal Liberty

Infringement on Personal Liberty


Overview of compulsory voting as an infringement of liberty. RMF Lacroix, Justine. "A liberal defence of compulsory voting." Politics 27.3 (2007): 190-195.
The main argument against compulsory voting hinges on the principle that individual liberty should prevail over other democratic ideals such as equality or participation. The refusal of compulsory voting is thus buttressed on the postulate that citizens must be free to decide if they choose to vote or not. All citizens should have the right to be apolitical, and the act of voting must remain a personal choice, and not an obligation. According to this view, the compulsory vote would be a case of strong paternalism. Strong paternalism occurs when there is intervention to protect or benefit a person, despite that persons informed and voluntary denial of consent to the paternalistic measures proposed (192)

Mandatory Attendance at polls is still Mandatory Voting AMS Fred L. Smith. A Case against Mandatory Voting. American Spectator. http://spectator.org/archives/2011/07/25/case-against-mandatory-voting
Ornstein was quick to point out that he doesnt necessarily support mandatory voting, but rather, in accordance with the system currently in place in Australia, mandatory attendance at the polls. To me, this is just an attempt to deflect attention from the mandatory part. Poll attendees are still required to cast a ballot, and in Australia those who fail to do so even if they showed up at the polls can be prosecuted. Even choosing none of the above or X, as is possible in Australia, involves casting a vote.

foundationbriefs.com

Page 45 of 104

Sept/Oct 2013

Neg: Infringement on Personal Liberty

A forced vote can contradict notions of self-interest and personal liberty, DAT 9/9 11:48pm Lever, Annabelle. Compulsory Voting: A Critical Perspective. British Journal of Political Science (2009).
People are not always good judges of their own interests, individually or collectively. Still, respect for peoples freedom and equality and capacities for reasoned judgement, generally tells against forcing them to exercise their rights, or to make the most of their liberties and opportunities. This is partly because there are normally other, less intrusive, ways to promote peoples self-interest than forcing them to act in a self-interested manner. But the difficulty with forcing people to pursue their self interest is, also, that informed, intelligent and reasonable people can disagree about the importance of self-interested ends relative to altruistic ones; and can disagree about the merits of self-interested behaviour in particular, as well as in general. Consequently, unless people have a duty to pursue their self-interest by voting, there is no justification for forcing them to vote, simply because voting would be good for them. Voting is not always in peoples interest. So even if people have a duty to vote when voting is in their selfinterest, we still need some reason to believe that voting is in their interests. This is less easy then we might expect. As voters, we can only protect our interests by choosing between the available political candidates or parties who solicit our vote. If none are in our interests, there is no self-interested case for voting. If they are all compatible with our interests, it may still not be in our interests to vote, even if some of these would be better at protecting our interests than others. Unless our vote is necessary to secure the election of the candidate that is best for us or to prevent the election of the one that is worst we may have no self-interested reason actually to go out and vote. This, of course, is why the rational choice literature insists that it is irrational to vote in circumstances where millions of otherwise reasonable men and women are clearly ready, even eager, to do so. So, even if we have a duty to promote our self-interest by voting, it is by no means clear that this translates into a duty to vote at most, let alone all, elections. Democratic voting rights protect our interests as individuals even when we do not exercise them. This, in part, is why it can be so important that people have legal rights to vote, whether or not they actually exercise them. In and of themselves, both moral and legal rights raise the threshold that arguments for coercion must leap in order to be justified. In this, the right to vote is no different from the right to marry: it protects our self-interest even where we do not exercise it, by ensuring that we are not married off against our will, or denied a voice in collectively binding decisions. In the case of the right to vote, it is only when used in coordination with strangers that its exercise is likely significantly to advance our interests over the baseline protection secured by its bare existence. By ourselves, however, we cannot ensure that others will be willing to cooperate and coordinate politically, and the effort to organize such cooperation may not be worth it. So even if having an equally weighted vote can be critical to our freedom and equality, it is an open question if and when its exercise will promote our interests.

foundationbriefs.com

Page 46 of 104

Sept/Oct 2013 Neg: Infringement on Personal Liberty Peoples self-interest, then, is unlikely to provide a justification for forcing them to vote, even though an important justification for democratic voting rights is that these are helpful, often necessary, to protect people. However, the difficulty with compulsory voting is more fundamental than that. To force people to vote, on paternalist grounds, is to suppose that the election of one of the candidates predictably threatens them with serious harms which they morally ought to avoid. But while democratic politicians pursue policies whose costs and benefits are unequally distributed, it will be hard to construe those unequal costs and benefits as constituting such harms at least as long as we suppose that the candidates/political parties from which we must choose our government are, genuinely, democratic. So, while we may have a duty to resist racist and undemocratic political candidates, and that this may require us to vote against them, a more general duty to vote on self-interested grounds is inconsistent with core assumptions about democratic politics.

Punishment is Undemocratic AMS Armin Shafer. Republican Liberty and Compulsory Voting. Max Planck Institute for the Study of Societies November 2011.
Finally, even if one accepts that voting is valuable to the political community, that most of the time a choice between different platforms is meaningful, and that individual abstention should not be allowed to become the general norm, it does not follow that coercion is justified against those who do not vote, especially, since it is not sufficient to make voting legally obligatory on paper, as recent empirical work has demonstrated. To be effective, punishment for non-voters cannot be trivial or merely symbolic.

Possibility of Jail Time for Non-Voters is Undemocratic, JMR Rovensky, Jan. Voting: A Citizens Right, or Duty? The Case against Compulsory Voting. Guido Carli Free International University for Social Studies. 2007. Pg 95.
Probably the strongest impact of compulsory voting laws is the possibility to end up behind bars as a result of non-voting. Though CV supporters point out that, for example in Australia, if someone does end up in jail, it is because they did not pay the allocated fine, not because they did not vote, to me this sounds as mere wordplay. The fact of the matter is that the principle remains the same, regardless for what reason the individual question is sent to jail his incarceration will somehow be connected to the fact of not voting. Lever (2007: 19) duly points out, that it is a predictable consequence of compulsory voting that people will go to prison, and end up with a criminal record, either because they cannot or will not pay the fine for non-voting. If we consider the case of Australia, after the election in 1993 at least 43 non-voters received a jail sentence as a result of this (Bennett 2005: 7); some cases make the news if the protagonists are vociferous enough. However, Australia is not the only country, where voters may end up behind bars: in Greece, Cyprus and the Philippines it is possible to be incarcerated as a direct consequence for non-voting.

foundationbriefs.com

Page 47 of 104

Sept/Oct 2013

Neg: Abstention Crucial to Democracy

Abstention a crucial act in democracy


Compulsory voting forces citizens into an unfair process which is potentially undemocratic, DAT Hanna, Nathan. An Argument for Voting Abstention. Public Affairs Quarterly. http://www.pages.drexel.edu/~nth34/Nathan_Hanna/Papers_files/voting.pdf
Citizens are often represented unevenly. For example, representation in the US Senate is significantly uneven and representation in the Electoral College is uneven to a lesser extent. Representation is typically distributed in a winner take all (majoritarian) fashion rather than proportionally. Voting districts are not determined by standardized means but are often gerrymandered for the express purpose of manipulating citizens relative degree of electoral influence. Access to the polls and the reliability of voting technology differs significantly. Certain citizens are barred from voting. For example, felons are barred from voting in several states. This is just a sample of characteristics that could reasonably be considered seriously unfair. Other things could be added to the list. For the sake of argument, I will grant that there is room for reasonable disagreement about how unfair these characteristics are and precisely how their alleged unfairness impacts electoral legitimacy, if at all. This means there is room for reasonable disagreement about their bearing on our obligations and permissions regarding voting. I will not argue that these characteristics are seriously unfair. But I do think it reasonable to hold that they are. This alone does not entail that it is reasonable to think that one has an obligation to abstain, however. Additional considerations are necessary: there is currently no reasonable prospect of correcting these unfair characteristics by voting and voting arguably perpetuates them by helping to confer a false appearance of legitimacy on the electoral process. Rather than vote in a US election, it may be the case that one should abstain because, given the nature of the electoral process and the prevailing electoral conditions, doing so would violate ones obligation not to vote badly. In the next section, I will consider several objections. Before addressing them, though, there is an important one to get out of the way. (5-6)

foundationbriefs.com

Page 48 of 104

Sept/Oct 2013

Neg: Abstention Crucial to Democracy

Abstention underlines the power of the individual over government. RMF Lever, Annabelle. 2009. Is Compulsory Voting Justified? Public Reason 1 (1): 57-74.
The right to abstain, or to refrain from political self-identification and participation is an important one, symbolically and practically. It captures two ideas that are central to democracy. The first is that government is there for the benefit of the governed, not the other way round. The second is that the duties and rights of citizens are importantly different from those of their representatives, because the latter have powers and responsibilities that the former do not. Citizens do not owe their government electoral support or legitimacy. (67)

Abstention is a Valuable and Powerful Political Tool , JMR Armin Shafer. Republican Liberty and Compulsory Voting. Max Planck Institute for the Study of Societies November 2011. Pg 14.
The second normative argument against compulsory voting sees abstention as a valuable political act in itself. If one dislikes the party platforms or the political personnel on offer, there might be political reasons to abstain. In fact, if participation in an election perpetu- ated injustice unfair electoral rules or a corrupt regime it would be morally appropri- ate to abstain (Hanna 2009). Not to vote under these circumstances can convey a stronger political message than voting for the lesser evil does. Stripping citizens of the possibility to send this message impoverishes democracy. Creating an artificially high turnout rate could cover up a lack of interest and the level of political dissatisfaction among the citi- zens (Franklin 1999: 206). If, in particular, the marginalized do not vote because they feel generally sidelined from society, unable to obtain access to education, decent housing, or jobs, it seems difficult to see
how compulsory voting will address, rather than exacerbate, the alienation of these non-voters, who are typically the objects, not the subjects, of political debate and policy, and who typically constitute the problems that politicians are competing to solve. (Lever 2008: 62)

From this perspective, making people vote appears to be a palliative move rather than an appropriate cure for the disease.

foundationbriefs.com

Page 49 of 104

Sept/Oct 2013

Neg: Voting Not a Duty

Voting not a duty


The value of voting can be reasonably contested. RMF Lever, Annabelle. 2009. Is Compulsory Voting Justified? Public Reason 1 (1): 57-74.
Still, it will not be easy to ground a general duty to vote on this natural duty, because in general it is unclear why support for just institutions should take the form of electoral participation, rather than anything else. Reasonable people can disagree about the value of political participation relative to other forms of social participation and support, and even those who value political participation may disagree about the value of voting, compared to other forms of political activity. So it is doubtful that the natural duty to support just institutions can justify legal duties to vote, even though it may sometimes give us morally compelling reasons to vote in some elections.

Living in a liberal democracy allows citizens to choose to avoid politics JMR Brennan, Jason. Polluting the Polls: When Citizens should Not Vote. Australasian Journal of Philosophy. Dec. 2009. Pg 544.
To live in a well-functioning liberal democracy is a great gift and something citizens should be thankful for. Yet one reason liberal democracy is such a great gift is that it does not require us to be political animals. It makes space for many ways of life, including avowedly non-political lives. In parallel, we might say that a good feature of well-functioning markets is that they make people rich enough to afford to engage in non-market activities and even in some cases to avoid the market altogether. A good liberal democracy would make people safe enough in their status as free and equal citizens that they could freely choose to avoid politics. Liberal democracy is an important public good. We should all do our part to maintain it. One way a person can do his part is by bowing out. A bad vote cancels a good vote. If a good vote is a gift to society, avoiding a bad vote is also a kind of gift. In fact, using Lomasky and Brennans formulae, we can construct scenarios under which avoiding a bad vote has the same expected value as a making a good vote.9

foundationbriefs.com

Page 50 of 104

Sept/Oct 2013

Neg: Voting Not a Duty

Voting is expressive, not decisive; therefore not a duty


The chance that a cast vote decides the election result is effectively zero. RMF Mackie, Gerry. "An Examination of the Expressive Theory of Voting." (2011).
An economist would be embarrassed to be seen at the voting booth (Dubner and Levitt 2005). Although most citizens vote, the standard view in social science is that its irrational to do so. Its extremely unlikely t hat any one vote would break a tie, and when a single vote does not break a tie it has nothing to do with the outcome. Since voting is costly, almost any single vote would be irrational. The paradox of nonvoting was first stated by Downs (1957, 244 - 246), and is often formulated as follows. B is the individuals Benefit from a winning election outcome, C is the Cost of the individual voting, and p is the Probability that an individuals vote is pivotal in causing the winning election outcome. An individual would vote then, when pB C > 0. The probability of being pivotal, however, is minuscule, effectively zero; for any individual, the act of voting is all cost and almost no benefit, and hence no one should vote. (2)

Voting is an expressive act, showing support for a candidate or issue. RMF Lomasky, Loren. DO VOTERS GET WHAT THEY WANT? Reason Magazine. November 1992.
I propose that voting, like cheering at a football game or sending a get-well card, should be understood as primarily an expressive rather than an instrumental activity. One votes for Alfred E. Neuman rather than Pat Paulsen as an act of expressing support for Neuman rather than as a deliberate attempt to raise the likelihood of Neuman's victory. Rational individuals will indeed vote provided that the value they assign to the expressive returns obtained through a vote is greater than the costs thereby incurred. Despite the exceedingly remote chance of swinging the election, voting is as rational as attending football games and cheering lustily for one's team (and sitting out an election is as rational as spending one's Saturday gardening if one prefers flowers to football). This expressive theory of voting avoids the schizophrenia of the standard self-interest theory of politics. Those who argue for compulsory voting are arguing that voting is required in order to decide an issue or election. Citizens ought to cast a vote to protect or advance their interests. Even implied in a duty to vote is that decisions must be made by the voter. But as these two pieces of evidence help to explain, that is not the reality of voting. Casting a vote does not decide the outcome of the election. Your one vote is too small. Instead, it is merely the expression of a person who gets utility from such an expression. It is therefore a personal act of consumption, not a public act helping society. Thus, such an act is outside the scope of governmental compulsion. Liberty should not be curtailed in the name of forcing people to make personal expressions on their attitudes about issues/candidates.

foundationbriefs.com

Page 51 of 104

Sept/Oct 2013

Neg: Voting Not a Duty

Democracy is CompetitiveNo Duty to Vote


Lever, Annabelle. "Liberalism, Democracy and the Ethics of Voting." London School of Economics
But is non-voting the equivalent of free-riding, or of unfairly seeking to benefit from the efforts and sacrifices of others? Political realism suggests that it is not. Whatever is wrong with not-voting, it cannot be that non-voters are selfishly exploiting the idealism, energy and public-spirited efforts of the BNP and their ilk. This is not because the latter are evidently more self-interested than other voters. Whether they are or not is an empirical question. The problem, rather, is that we are entitled to refuse, and actively to oppose, the benefits that the BNP seeks to promote. Non-voters, then, are not exploiting the BNP. Nor are they exploiting selfinterested voters, however respectable and democratic the parties for which they voted. It is not obvious, either, that they are exploiting altruistic voters simply because they are not helping them. So, reflection on how and why people vote casts doubt on the idea that non-voters are selfishly preying on the publicspirited efforts of voters. (Lever 2009 a,b) The idea that non-voters are free-riders, assumes that voting is a collective good whether because high levels of turnout are necessary to democratic legitimacy or for some other reason. But this begs the question, whether high levels of turnout are a collective good. Turnout has partisan effects. So even if some level of turnout is a public good, voting is not a pure public good as long as it has some bearing on who wins or loses an election. To suppose that people are morally wrong to abstain, therefore, requires us to assume that the cooperative aspect of voting is more important than the competitive. This is not a conceptual truth about elections, and may be false empirically. (Lever 2009 a,b) We cannot evade the complexity of democratic politics and morality, then, by insisting that democratic elections are a public good. Indeed they are. But this no more requires us to vote than it requires us to join a political party or to stand for election ourselves. A sufficient range and quality of parties and leaders is a prerequisite for democratic legitimacy and, off-hand, seems at least as important as ensuring a sufficient quantity and quality of voter participation. Moreover, morality sometimes requires people to assume positions of leadership and responsibility that they would otherwise choose to forego. Nonetheless, it is incredibly difficult to get from the idea that we may sometimes have such duties to the conclusion that we actually do have such duties. What we are morally required to do in politics depends importantly on what other people do, what they are likely to do, and what they are entitled to do. Hence the complexities of democratic politics and morality. Political scientists, historians, novelists, playwrights and politicians have done an enormous amount to clarify that complexity. Political philosophers have much to learn from them.

foundationbriefs.com

Page 52 of 104

Sept/Oct 2013

Neg: Costs of Mandatory Voting

Costs of Mandatory Voting


Mandatory voting artificially imposes disproportional costs, DAT Downs, Anthony. An Economic Theory of Democracy. New York: Harper and Row, 1957.
Heretofore we have assumed that voting is a costless act, but this assumption is self-contradictory because every act takes time. In fact, time is the principal cost of voting: time to register, to discover what parties are running, to deliberate, to go to the polls, and to mark the ballot. Since time is a scarce resource, voting is inherently costly. This fact alters our previous conclusion that everyone votes if he has any party preference at all. When there are costs to voting they may outweigh the returns thereof; hence rational abstention becomes possible even for citizens who want a particular party to win. In fact, since the returns from voting are often miniscule, even low voting costs may cause many partisan citizens to abstain. The importance of their abstention depends on the effects it has upon the distribution of political power. Such effects can stem from two sources: (1) biases in the distribution of ability to bear the costs. By real returns we mean those which each citizen would perceive in a perfectly informed world of voting, and (2) biases in the distribution of high returns from voting. The only direct money costs connected with registering to vote and voting are any poll taxes extant and the cost of transportation. Ability to bear these costs varies inversely with income, so upper- income citizens have an advantage. Where poll taxes do not exist, the principal cost of voting is usually the utility income lost by devoting time to it rather than something else. If the time must be taken out of working hours, this cost can be quite high, in which case high-income groups again have an advantage. But if the time comes during leisure hours, there is no reason to suppose any such income-correlated disparity exists. At first glance, all of these costs may appear trivial, and biases in ability to bear them seem irrelevant. However, the returns from voting are usually so low that tiny variations in its cost may have tremendous effects on the distribution of political power. This fact explains why such simple practices as holding elections on holidays, keeping polls open late, repealing small poll taxes, and providing free rides to the polls may strikingly affect election results. (6-7). The cost to any act voting is, at its least, time. And the difference between compulsory voting and a similar compulsory act (jury duty) that also has a cost of time in return for what is considered a civic duty is that compulsory voting is no sense coded into the constitutions of some democracies (e.g. the United States) or even enforced in others.

foundationbriefs.com

Page 53 of 104

Sept/Oct 2013

Neg: Costs of Mandatory Voting

Costs are not Trivial AMS Fred L. Smith. A Case against Mandatory Voting. American Spectator. http://spectator.org/archives/2011/07/25/case-against-mandatory-voting
Ornstein eventually conceded that compulsory voting would require trivial enforcement costs and would constitute a trivial loss of freedom. Yet the cumulative impact of past trivial costs has created todays huge budget deficit. And even trivial losses of freedoms over time move us in the direction of tyranny. Opinions may differ on whether greater voter turnout is a good thing, but no one should support policies designed to force people to be free.

foundationbriefs.com

Page 54 of 104

Sept/Oct 2013

Neg: Voting Against Own Interests

Voting against own interests/ideology


Uneducated voters who are forced to vote end up voting against own interests. RMF Selb, Peter, and Romain Lachat. "The More, the Better? Counterfactual Evidence on the Effect of Compulsory Voting on the Consistency of Party Choice." European Journal of Political Research (2009). Web.
Turning now to these variances, we can see that the choices of voters who declared that they would never vote again if voting were voluntary are about three (among Flemings) to six times (among Wallonians) as variable as the party choices of the voluntary voters, with their proximity-based utilities being held constant. In other words, the proximity model indicates that unwilling or ignorant voters forced to the polls by CV tend to make choices that are considerably less consistent with their policy preferences than voluntary voters. (587) This study examined Belgian voters.

People cannot be expected to vote in an objective or well-informed manner, DAT Goldman, Alvin I. Why Citizens Should Vote: A Causal Responsibility Approach. Social Philosophy and Policy Foundation, 1999.
First let us ask what might be meant by the phrase "objectively best candidate." Elsewhere I have suggested that if candidate A would produce a set of outcomes higher on the preference-ordering of a majority of citizens than the set of outcomes candidate B would produce, then A is a democratically better candidate than B. This is one possible way to give content to the phrase "objectively best candidate," though others, of course, might be proposed. Now a voter might be said to have objectively good reasons to vote for a certain candidate if that candidate is objectively best. But what if a citizen does not know, and indeed has no idea, which candidate is objectively best? Should such a citizen still be encouraged to vote rather than abstain? On the approach I favor, citizens should not be encouraged to vote, full stop. Instead they should be encouraged first to gather enough information and then to vote. The point of becoming informed, of course, is to increase the probability of making a good choice, that is, of choosing the objectively best candidate. The upshot is that voting is not necessarily and without qualification a desirable or dutiful act. Consider an uninformed citizen, late on election day, who has no time to become informed before the polls close, but wonders whether he should vote. The present approach would not justify his voting. In this respect, the present rationale differs from both the Kantian and expressivist approaches, which presumably urge people to vote under all circumstances. I do not regard it as a defect of the current rationale that it has this qualified aspect. I am unconvinced that a person ought to vote, or has a duty to vote, even when he is both uninformed and no longer has time to become informed. (209)

foundationbriefs.com

Page 55 of 104

Sept/Oct 2013

Neg: Voting Against Own Interests

A dearth of informed voters turns elections into exercises of random probability, DAT Jakee, Keith, and Martin Kenneally. Increasing Voter Turnout: What Difference Can Compulsory Voting Make? Florida Atlantic University, 2009.
If abstainers are less interested and less informed about the political process, as per Jakee & Sun (2006), Lacy and Burden (1999), or Lassen (2005), then it is even less clear that increasing their presence in the election is a desirable objective. The reason is that their votes can effectively be treated as random (Jakee and Sun 2006) and, if this is an accurate assumption, forcing them to vote causes the CVR outcome to become increasingly random, as the percentage of involuntary voters (P3) increases. Indeed, this specific scenario of a very high abstention rate and low voter turnout is one, according to our analysis, in which a CVR can make a difference compared to a VVR. However, implementing a CVR in this case may turn the election outcome into something akin to a coin toss because of the large percentage of purely random votes. Determining an election by a random coin toss surely implies an electoral result that is wholly unreflective of any underlying community preferences further weakening the argument in favor of imposing a CVR. A CVR is a compulsory voting rule; a VVR is a voluntary voting rule. The authors of the study used statistical analysis of multiple election scenarios, altering variables for turnout, whether a seat was safe, etc.

Citizen has a Responsibility Not to Vote Poorly JMR Brennan, Jason. Polluting the Polls: When Citizens should Not Vote. Australasian Journal of Philosophy. Dec. 2009. Pg 538.
In some elections, it will be difficult even for highly educated experts to judge the expected consequences of electing one candidate over another. Judging candidates comparative merits is often, but not always, difficult even for experts. Provided that the evidence shows that each candidate is likely to be on the whole good rather than harmful, then well-informed, adequately rational, just voters can be said to vote well regardless of which candidate they select. The claim that voters ought not to vote badly does not imply the stronger claim that they must vote only for the most optimal candidate. The most common forms of bad voting are voting 1) from immoral beliefs, 2) from ignorance, or 3) from epistemic irrationality and bias.
As proven in the three previous pieces of evidence when citizens are forced to vote, oftentimes they vote ignorantly or from epistemic irrationally and bias, which Brennan defines as bad voting and argues that it is the citizens responsibility to refrain from bad voting.

foundationbriefs.com

Page 56 of 104

Sept/Oct 2013

Neg: No Change in Outcome

Compulsory voting does not change outcome


Removing compulsory voting does not change the outcome of an election. RMF Selb, Peter, and Romain Lachat. "The More, the Better? Counterfactual Evidence on the Effect of Compulsory Voting on the Consistency of Party Choice." European Journal of Political Research (2009). Web.
Wherever these differences come from, we observe that voters prone to abstain if CV were abolished are less inclined to make their decisions in a way that coherently reflects their issue preferences. Yet do their party choices also systematically differ from those of the voluntary voters? In other words, would election results change in a predictable direction if these voters actually could abstain? Moreover, what would election results look like if these voters chose parties on issue grounds in the same way as did the voluntary voters? As to the first counterfactual, we find barely any differences between the results. Obviously, the self-declared abstainers do not systematically differ in their party choices from the voters who expressed more willingness to participate under the voluntary vote. Thus, frequently expressed concerns that the left parties, in particular, would suffer electoral losses if CV were abolished are not supported by our data. Likewise, our second counterfactual suggests that election results would not differ dramatically if the probable abstainers voted as coherently in accordance with their policy preferences as did the voluntary voters. (589-590) These researchers used poll data from Belgium, where compulsory voting was in place for a 1995 election. Polling of voters identified who would vote regardless of the compulsory voting law, and who would not vote if not forced to do so. By filtering these groups of people, the researchers found that the election results would likely be similar regardless of the compulsory voting law.

foundationbriefs.com

Page 57 of 104

Sept/Oct 2013

Neg: No Change in Outcome

The underlying problems and inequalities of the voting process are not eliminated, DAT Jakee, Keith, and Martin Kenneally. Increasing Voter Turnout: What Difference Can Compulsory Voting Make? Florida Atlantic University, 2009.
Our results suggest that, in these circumstances even with low turnout rates, moving from a voluntary to a compulsory system is likely to have only have a minor impact at most: safe seats will remain safe and beyond the reach of challenger candidates, even under a compulsory system. And, in Northern European countries, where turnout is already very high, CVRs are likely to have even less impact on seat turnover because the pool of abstainers upon which the trailing candidate must draw is small. Consequently, those election observers who are dissatisfied with current low rates of voter turnout may have to look to institutional changes other than compulsory voting to change what they view as unrepresentative election outcomes. Other measures to make seats more representative might include redistricting, or institutional changes, such as term limits, campaign spending reform, or rules governing the incumbent advantages of pork barrel spending. We are not actively endorsing these changes, since attempts to make seats more representative in the context of redistricting congressional boundaries, for example, comes with an additional set of problems (Buchler 2007). In any case, compelling more citizens to vote is unlikely to have to the effects desired by those who lament low voter turnout rates. Furthermore, if a CVR is unlikely to have little effect on VVR results, it is reasonable to question the value of imposing the costs of compulsion on both the abstainers and the electoral commission responsible for detecting and enforcing a compulsory vote. (24-25) CVR is a compulsory voting rule; a VVR is voting rule.

foundationbriefs.com

Page 58 of 104

Sept/Oct 2013

Neg: No Change in Outcome

Increase in Voter Turnout due to CV would not make a difference in Elections, JMR Armin Shafer. Republican Liberty and Compulsory Voting. Max Planck Institute for the Study of Societies. November 2011. Pg 13.
Studies that compare voters and non-voters are usually based on survey analysis.9 For example, Heighton and Wolfinger (2001) use survey data from US presidential elections in 1992 and 1996 to see how voters and nonvoters differ. While they find that non-voters are slightly more favorable toward liberal (social democratic) policies and, in 1996, expressed greater support for Bill Clinton, the differences in relation to most other items are rather small. They conclude that non-voters do not form a homogeneous group and that an increase in turnout would hardly make much of a difference. Simulating how Senate election results would have changed had everyone voted, Citrin et al. (2003) also conclude that those who abstain are more likely to favor the Democratic Party, but that few results would have been different with higher turnout rates.10 In simulating which party non-voters would have chosen had they voted in the European Parliament election of 2009, van der Eijk et al. (2010) also conclude that the effects would have been negligible for the vast majority of parties. Finally, Kohler (2011) shows that higher turnout would only have affected government formation in one German general election (2005) with a reasonably high level of probability.

foundationbriefs.com

Page 59 of 104

Sept/Oct 2013

Neg: Undermines Legitimacy

Undermining the Legitimacy of Voting


Compulsory voting institutionalizes complacency with flaws in the electoral process, DAT Hanna, Nathan. An Argument for Voting Abstention. Public Affairs Quarterly. http://www.pages.drexel.edu/~nth34/Nathan_Hanna/Papers_files/voting.pdf
Within the context of a seriously unfair electoral process and political conditions that limit the prospects of correcting the unfairness by means of voting, however, one need not see this opportunity as having much worth. The unfairness of the electoral process and the conditions in which it is embedded arguably causes it to consistently present citizens with inadequate choices and to present them in ways that give some an unfair advantage in shaping government. A common criticism of majoritarian electoral processes in the US context, for example, is that they make it excessively difficult for third parties to compete and to voice their ideas in the electoral arena. This forces many voters to either vote for viable candidates who would represent their views relatively poorly or for unviable candidates who would represent their views better. Faced with such choices, abstention need not be seen as wasting a valuable opportunity. In the absence of countervailing conditions like a close race that threatens the election of a horrible candidate, abstaining need not be seen as incurring a significant personal cost in the context of such a process. But also and this gets at another important error behind many anti-abstention views abstention need not be a refusal to influence government. Rather, it can be an attempt (or part of one) to influence things in another way, a way outside the narrow confines the electoral process conventionally affords us. It can be a way of trying to influence the electoral process itself and others attitudes toward it. Abstaining can express dissatisfaction with the electoral process and can be a way of advocating reform (cf. Brennan and Lomasky 2000: 83-84, Goldman 1999: 215). It can be an act of protest. Such acts are not causally impotent. They have an influence on policy all their own. They may not achieve the desired result, but in that respect they are similar to voting. And the desired result a more legitimate and fairer electoral process is extremely valuable. Part of the hope is that a better process would present citizens with more worthwhile choices. The fact that many do see voting as a valuable opportunity can make abstention an especially powerful form of protest. At the very least, abstaining can be a refusal to help perpetuate the serious problems that need reform. There is an implicit notion that is power is inherent to only voting. Hanna essentially argues that abstention is in itself a voteone for an institutional reform, rather than a candidate.

foundationbriefs.com

Page 60 of 104

Sept/Oct 2013

Neg: Undermines Legitimacy

Universal Voting can pollute the democratic system JMR Brennan, Jason. Polluting the Polls: When Citizens should Not Vote. Australasian Journal of Philosophy. Dec. 2009. Pg 548
I see myself as a defender of democracy. I wish to keep the voting process free of pollution, and what defender of democracy wishes to see her favored system polluted? Many democrats are concerned both with democratic procedures and democratic outcomes [Christiano 2004; Brettschneider 2007]. Not just any outcome produced by democratic procedure is acceptable, nor is every outcome aligning with democratic values acceptable regardless of what procedure produced it. Universal voting by bad voters might make procedures more democratic than massive abstention by people who would vote badly. Yet, this does not mean the outcome of this procedure will be align better with democratic values, and thus does not mean that opposing universal voting is inherently undemocratic. When people call for universal or extended participation, we have to ask what would be the point of the institution of universal participation. If we are passionate lovers of democracy, we might celebrate what universal participation would symbolize. Yet, in the real world, we have to ask how institutions would function. Institutions are not people. They are not ends in themselves. They are not paintings, either, to be judged by their beauty, by what they symbolize, or who made them. Institutions are more like hammersthey are judged by how well they work. Good institutions get us good results; bad institutions get us bad results.

foundationbriefs.com

Page 61 of 104

Sept/Oct 2013

Neg: Decrease Quality of System

Mandatory Voting would Decrease Quality of System


Those who Do Not Want to Vote Do Not Make Good Voters AMS Armin Shafer. Republican Liberty and Compulsory Voting. Max Planck Institute for the Study of Societies November 2011.
Low rates of participation, however, might not just be irrelevant for the quality of democracy but actually be a way to improve it. Since non-voters tend to be less well-informed and less interested in politics, it might be a blessing in disguise if they stayed at home on Election Day (Rosema 2007). Voluntary voting separates the truly interested from the ill-informed and confused, who hold misguided opinions about politics (McClosky 1964: 376). Taking this argument one step further, Brennan (2009) argues that people who are likely to vote badly because they are uniformed or ignorant have a moral duty not to vote since they would pollute the polls. Caplan (2007: 198) lends support to this idea, as he finds that many citizens hold irrational beliefs about the economy, i.e., their opinions deviate from those of PhD students in economics. Since economic literacy, so constructed, rises with formal education, low and uneven turnout is advantageous because in this case the median voter will be more economically competent than the median citizen. Efforts to increase voter turnout would harm the epistemic quality of democracy.

Random Voting delegitimizes a democracy JMR Gratschew, Maria. Compulsory Voting In Western Europe. International Institute for Democracy and Electoral Assistance. 2004 pg 30.
Another consequence of compulsory voting is the possible high number of 'random votes'. Voters who are voting against their free will may tick off a name at random, particularly the name at the top of the ballot paper. The voter does not care for whom he or she votes as long as the government is satisfied that they have fulfilled their civic duty. What effect does this immeasurable category of random votes have on the legitimacy of the democratically elected government?

foundationbriefs.com

Page 62 of 104

Sept/Oct 2013

Neg: Decrease Quality of System

Compulsory does not address the problems of the voting process, DAT Verba, Sidney. Thoughts About Political Equality. Harvard Kennedy School of Government, 2001. Web.
True political equality, where all ordinary citizens (i.e., those not in governmental decision making positions) have equal influence, would be impossible to attain and probably very bad. In considering the data on political equality and how one might respond to those data, it is important to keep this in mind. In brief form, here are some reasons why that is the case. Achieving actual equal voice would be very difficult, involve severe governmental intervention, and require limitation on freedom. It could be achieved by putting a floor and a ceiling on activity. A floor would require all to be active at some defined level -- making voting compulsory or attendance at political meetings or political contributions compulsory. Voting is compulsory (usually with minor penalties for failure to vote) in some countries. It is innocuous, perhaps, but would go against the grain in America and not add much to political equality. Compulsory attendance and contributions smacks much more of authoritarian techniques than democratic ones. Ceilings are difficult: A ceiling on political activity exists in relation to the vote -- one person, one vote. But ceilings on money have run up against court interpretations of the First Amendment, and any limitation on other activities -- on writing letters, protesting, attendance at meetings -- would certainly do so. (1)

foundationbriefs.com

Page 63 of 104

Sept/Oct 2013

Neg: Decrease Quality of System

Under a democracy, the responsibility of voting is different from the requirement to vote, DAT Moraro, Piero. Why Compulsory Voting Undermines Democracy. Living Ethics, 88. St. James Ethics Centre. Winter 2012.
The main argument in defence of compulsory voting is the importance of raising citizens participation in elections. Australias turn out rate has never fallen below 90% since 1924, in striking comparison with most Western countries that struggle to reach 60%. These are nevertheless, merely quantitative considerations: the fact that more people go to vote is not a better thing for democracy. In fact, it could be argued that compulsory voting is likely to do more damage than good, by reducing the quality of the electoral outcome. Forcing everyone to vote means that the voice of those with no interest in politics will influence the decision about who rules the country. This generates what author Jason Brennan calls pollution of the polls in his book The Ethics of Voting and is one of the main causes of the actual crisis of democracy worldwide: incompetent politicians winning elections through media control (the recent case of Italy under Silvio Berlusconi epitomises this phenomenon). By the same token, compulsory voting cannot be defended by arguing that a governments legitimacy of a majority formed by a low turn-out is questionable, for numbers alone do not add credibility in this regard. Favouring democracy to other forms of government cannot be because of the mere fact that democracy allows everyones voice in the public arena. This might be, if anything, a reason against democracy. Dragging people to the polls will do nothing to improve the quality of our democratic lives, insofar as people do not take seriously what they are doing. There must be something else that makes democracy preferable to other forms of government. This something else is the empowerment of the citizens. In a democracy, everyone has the power to partake in determining the rules according to which the community should be run. However, the misunderstanding lies in the fact that democracy does not entitle citizens to do everything: it confers not only entitlements but also responsibilities. Those who do not care about politics should not vote. (1)

foundationbriefs.com

Page 64 of 104

Sept/Oct 2013

Neg: Decrease Quality of System

Disenfranchisement as a Punishment Undermines Democracy JMR Rovensky, Jan. Voting: A Citizens Right, or Duty? The Case against Compulsory Voting. Guido Carli Free International University for Social Studies. 2007. Pg 95.
Another ghastly implication of a CV regime is disenfranchisement, as is the case in Belgium. Though admittedly the loss of ones voting rights is not permanent, I still feel especially strong about this case because I cannot see how a democratic country could have such provisions when the right to vote should be sacrosanct. This is a direct violation of the right to vote, putting the non-voter on par with convicted felons who in some states lose this right because of their crimes against the society. Trying to rebuff such a comment, as I suspect CV supporters would do, by stating that chronic non-voters would not really care if they lost their right to vote is simply irrelevant, because their crime is not such as it requires the state to infringe on one of the basic political liberties an individual has within a democracy. I therefore find the following Birchs (2007: 11) claim rather disturbing: If voting is considered to be a civic duty, it would seem logical that non-voters should have their civic rights restricted. The most obvious right to restrict is the right to vote itself. I deeply question such logical thinking. How does the disenfranchisement of the non-voter help the democratic system? Is the system more democratic and healthier by actually curtailing the universal right to vote? I seriously doubt this (Hill (2002c: 4-5) also seems to argue against disenfranchisement).

foundationbriefs.com

Page 65 of 104

Sept/Oct 2013

Neg: Voting vs. Turnout

Compulsory Voting vs. Compulsory Turnout


Lever, Annabelle. "Liberalism, Democracy and the Ethics of Voting." London School of Economics
Proponents of compulsory voting generally believe that people are morally obliged to vote unless they have conscientious objections to voting. No one thinks that there is a moral duty simply to turn out and tick your name off a list at election time unless people have a duty to vote. So, such justification as there is for compulsory turnout is parasitic on the justification of compulsory voting. It is therefore wrong to suppose that it is easier to justify compulsory turnout than compulsory voting. How compulsory voting is supposed to fix the problems of low and unequal turnout at elections is reasonably clear. (Lijphart, 1997; Lever 2009a) By contrast, it is unclear how compulsory turnout is going to solve these problems. Consequently, it is harder, not easier, to justify compulsory turnout than compulsory voting once we have allowed that people with conscientious objections to voting should be exempt from moral and legal duties to vote. (Pg. 2) This card could prove very valuable to the Negative in instances where the Affirmative attempts to claim both the benefits of compulsory voting as well as avoid the costs of compulsory voting, namely forcing people to actually vote. The point here is that if you are only requiring people to turnout then the benefits such as decreased inequality are not guaranteed. To obtain those benefits, the disenfranchised must actually vote however, doing so comes with its own set of costs (see Neg evidence from previous sections) that you, as the negative, should not let go ignored in round.

foundationbriefs.com

Page 66 of 104

Aff Counters

foundationbriefs.com

Page 67 of 104

Sept/Oct 2013

Aff Counters: No Good Alternative

Alternatives do not address the problem entirely


Even the full confluence of alternatives is not as effective as compulsion, DAT Hill, Lisa. Compulsory Voting in Australia: A Basis for a Best Practice Regime. Federal Law Review, Vol. 32.
Critics of compulsory voting sometimes suggest that low turnout can be ameliorated by other less coercive means, and a number of countries have sought alternative solutions, with varying degrees of success. In the United States, for example, a recent attempt to improve turnout and close the widening SES gap failed. Even though the National Voter Registration Act of 199324 ('Motor Voter Act') was carefully targeted at the most deprived and politically inactive members of the electorate, depressingly, and despite all hopes to the contrary, voting rates among registered voters actually declined. Other institutional means by which to improve turnout include: reducing the frequency and complexity of elections to address the problem of voter fatigue, moving election day to a Saturday or else declaring it a national holiday, providing for automatic registration and absentee voting, siting polling places in shopping malls to reduce the opportunity costs of voting, extending voting over two or more days or else offering voters incentives to vote. Making elections more salient (ie visible, consequential and therefore meaningful) to voters also seems to enhance turnout. One common source of low electoral salience is the choice-restricting effect of a simple plurality system, leading to depressed turnout and tactical voting. Accordingly, proportional representation is often proposed as a means by which to encourage voting. The adoption of all measures known to increase voting participation would be extremely difficult. But, if they were adopted, 90 per cent turnout rates in places like Britain, the US and Canada would be a real possibility. And yet there is one simpler solution that can raise turnout higher than all these reforms put together: compulsory voting. In fact compulsory voting is the only institutional mechanism that can achieve turnout rates of 90 per cent and above on its own. This has been verified in a number of countries in which compulsory voting has been adopted and, more importantly, systematically administered. Whereas turnout in most industrialised democracies is steadily declining, Australia has maintained healthy levels (around 83 per cent of the voting age population ('VAP') and 95 per cent of registered voters ('RV')) for many years. Belgium enjoys turnout of around 91 per cent RV, while in The Netherlands, for the 53 years that compulsory voting was in force (191770), turnout was consistently above the 90 per cent mark. Even those who do not necessarily approve of mandatory voting admit that it works. (484-485)

foundationbriefs.com

Page 68 of 104

Sept/Oct 2013

Aff Counters: Political Parties Dont Help

Political Parties do not Help Minorities Express Views


The Problem with Political Parties AMS Alex White. June 14, 2013. New Matilda. https://newmatilda.com/2013/01/14/defencecompulsory-voting
You could rely on the political parties to mobilise supporters to get to the polls to vote. Unfortunately, as we see in other countries, this largely results in a vocal minority dominating the public discourse (witness the Tea Party in the USA). Because the motivated minority is more likely to vote and to dominate the debate, it is actually in their interest to try to depress the turnout of opposition groups. We've seen this with the voter suppression laws that Republicans introduced in many states in the USA.

Interests of Political Parties Conflict with Interests of Democracy AMS Eric Liu. Why Voting Should Be Mandatory. Times. August 21, 2012. http://ideas.time.com/2012/08/21/should-voting-be-mandatory/
Some Republicans will oppose mandatory voting for the reason they now push voter IDs: to win. (Conventional wisdom says the more people who vote, the worse the GOP does). But if a tactic of disenfranchisement and electorate-amputation makes sense for the party (which is debatable), it is terrible for the country. As former director of the Office of Management and Budget Peter Orzsag has pointed out, we cant know what the ultimate partisan impact would be. One day Republicans could benefit.

foundationbriefs.com

Page 69 of 104

Sept/Oct 2013

Aff Counters: CV Does Change Outcomes

CV does change election outcomes


Methodology: author compares election results before and after CV across Australian states. Fowler, Anthony. "Electoral and Policy Consequences of Voter Turnout: Evidence from Compulsory Voting in Australia." Quarterly Journal of Political Science 8.2 (2013): 159-182.
In order to test the effect of compulsory voting on election results, I have collected the results of every state legislative assembly election from 1910 to 1950.5 The six Australian states provide an excellent opportunity to test for the effects of near-universal turnout because they adopted compulsory voting at different times. Employing a differences-in-differences design, I estimate the effect of compulsory voting on three different dependent variables: voter turnout, Labor Party vote share, and Labor Party seat share. As compulsory voting is implemented in one state, we can compare that states changes in voting behavior with the changes of other states at the same time. (168-169)

CV changed policy attitudes as well as policy itself. RMF Fowler, Anthony. "Electoral and Policy Consequences of Voter Turnout: Evidence from Compulsory Voting in Australia." Quarterly Journal of Political Science 8.2 (2013):
The significant effects of compulsory voting on election results suggest that compulsory voting may have influenced public policy as well. The Labor Party held systematically different positions than the other parties over many issues including unions, social spending, immigration policies, and the size of government. Therefore, a greater presence of Labor members in state assemblies and the national Parliament may have had significant effects on the lives of Australians at the time. Moreover, the new wave of workingclass voters may have led all parties to shift in a progressive direction. A historical account of Australias national politics around this time period suggests several potential effects of compulsory voting on public policy. The majority partys platform changed dramatically after the adoption of compulsory voting for federal elections in 1924. In the 1922 election the Nationalist Party maintained power, announcing a conservative domestic policy: First and foremost, we are against class legislation and class government (Hughes, 1922). In 1925, the first election under compulsory voting, the Nationalist Party maintained power but shifted dramatically on these issues: It has to be recognized that even under the conditions existing in Australia, the wages of our workers are not sufficient to enable them to safeguard against these evils [sickness, unemployment, and old-age] (Bruce, 1925). In just a three-year span, the majority party made a clear turn toward progressive domestic policies, and the introduction of more working-class voters into the electorate and increased presence of the Labor Party may have provided impetus for such a change. (173174)

foundationbriefs.com

Page 70 of 104

Sept/Oct 2013

Aff Counters: CV Does Change Outcomes

Compulsory voting significantly affected the share of votes going to the Labor Party. RMF Fowler, Anthony. "Electoral and Policy Consequences of Voter Turnout: Evidence from Compulsory Voting in Australia." Quarterly Journal of Political Science 8.2 (2013): 159-182.
Table 2 presents the results of all three difference-in-differences regressions along with the three corresponding models with state-specific trends. Column 1 shows that the implementation of compulsory voting increased turnout by 24 percentage points on average, from 67 to 91 percent. Columns 3 and 5 show that compulsory voting increased the Labor Partys vote share and seat share by 9 and 7 percentage points, respectively. These results are both substantively and statistically significant. Since the Labor Partys average seat share was 43 percent before the adoption of compulsory voting, these effects were meaningful enough to change the control of power in many legislative assemblies. Columns 2, 4, and 6 present the robustness tests allowing for state-specific trends and all three results are virtually unchanged. These additional tests lend credibility to the parallel trends assumption, because relaxing the assumption is inconsequential for the subsequent estimates. According to these results, average turnout in the absence of compulsory voting during this period would have been 67 percent, and the Labor Party would have received 44 percent of the votes. However, average turnout with compulsory voting would have been 91 percent, and Labor would have received 53 percent of the votes. This suggests that 78 percent of the new (marginal) voters brought to the polls by compulsory voting supported the Labor Party compared to just 44 percent of older (regular) voters.6 This estimate is substantively large but quite plausible, given what we now know about turnout inequality prior to compulsory voting. (171-172)

Other phenomena are unlikely to have caused this change. RMF Fowler, Anthony. "Electoral and Policy Consequences of Voter Turnout: Evidence from Compulsory Voting in Australia." Quarterly Journal of Political Science 8.2 (2013): 159-182.
After numerous discussions with Australian historians, I am unaware of any confounding variable which could have simultaneously led to compulsory voting and an increase in support for the Labor Party. (178)

foundationbriefs.com

Page 71 of 104

Sept/Oct 2013

Aff Counters: CV Does Change Outcomes

Unequal Turnout Equates to Unequal Influence JMR Lijphart, Arend. Unequal Participation: Democracys Unresolved Dilemma. The American Political Science review, Volume 91, Issue 1. March 1997. Pg 4-5
Nonvoters who are asked their opinions on policy and partisan preferences in surveys are typically citizens who have not been politically mobilized, and who, in terms of social class, have not developed class consciousness. It is highly likely that, if they were mobilized to vote, their votes would be quite different from their response in opinion polls Furthermore, the few studies that attempt the difficult task of directly testing the link between voter turnout, on the one hand, and tax and welfare policies on the other, all find compelling evidence that unequal voting participation is associated with policies that favor privilege voters over underprivileged nonvoters (Hicks and Swank 1992; Hill and Leighley 992; Leighley 1995, 195-6; Mebane 1994). Finally, perhaps the most persuasive evidence is the strong and direct link between turnout and support for left-of center parties found by Alexander Pacek and Benjamen Radcliff (1995). They analyzes all national elections in 19 industrial democracies from 1950 to 1990 and found that, as hypothesized the vote for left parties varied directly with turnout: The left share of the total vote increases by almost one-third of a percentage point for every percentage point increase in turnout. In short, the overall weight of the evidence strongly supports the view that who votes and how people vote matter a great deal. Indeed, any other conclusion would be extremely damaging for the very concept of representative democracy.

foundationbriefs.com

Page 72 of 104

Sept/Oct 2013

Aff Counters: No Threat to Personal Liberty

It Does Not Threaten Personal Liberty


JS Mill, father of liberalism, saw participation as necessary for successful governance. RMF Lacroix, Justine. "A liberal defence of compulsory voting." Politics 27.3 (2007): 190-195.
In his analysis of the best form of government, Mill clearly opted for a form of participatory democracy: it is evident that the only government which can fully satisfy the exigencies of the social state is one in which the whole people participate (Mill, 1999, p. 234). Such a conviction led him to promote extended suffrage on the basis that the rights and interests of every person are only secure from being disregarded when the person interested is himself able, and habitually disposed, to stand up for them (Mill, 1999, p. 224). (191)

Concept of equal liberty supports CV. RMF Lacroix, Justine. "A liberal defence of compulsory voting." Politics 27.3 (2007): 190-195.
Conversely, many other liberal thinkers argue that this [liberty as defined as free interprise] is an improper interpretation of the word liberty as this restrictive conception actually concerns the liberty of a few and thus implies constraints for the majority. A much more attractive ideal would be liberty for all ... in other words, the liberal commitment for liberty has resources that may be opposed to the libertarianism of the economic conservatives (Waldron, 1987, p. 129). That is the reason why the liberal commitment to liberty has been reformulated as a commitment for equal liberty, a principle that justifies solidarity policies which do not infringe on individual rights as they aim at guaranteeing liberty for all and creating the necessary conditions for the full exercise of individual liberty. The same argument can be used to defend compulsory voting. By encouraging all citizens, even the least motivated among them, to be informed and voice their opinions, compulsory voting would partially thwart the strong social determinants and oblige political parties to pay heed to the more marginalised electors. (194) This argument rests on the observations that without compulsory voting, the least economically, educationally, and socially advantaged people are underrepresented in elections.

foundationbriefs.com

Page 73 of 104

Sept/Oct 2013

Aff Counters: No Threat to Personal Liberty

The liberty of the least privileged must be guaranteed, CV does this by ensuring they have a voice in elections. RMF
As in John Rawlss model the only acceptable forms of inequalities are those that are beneficial to the least privileged part of the population, it is all the more difficult to contend that the recorded inequalities in electoral participation may serve the interests of this category of individuals. From this approach, the defence of compulsory voting echoes Shklars observations on democracy. As emphasised by Paul Magnette, in Shklars definition of liberalism ... it is first and foremost the liberty of the weakest that is protected by democracy. It may not make citizens equal, but at least it erodes the submission of the weakest (Magnette, 2006, p. 93).(194)

The Meaning of Individual Liberty AMS Alex White. June 14, 2013. New Matilda. https://newmatilda.com/2013/01/14/defencecompulsory-voting
Finally, the argument that individual liberty is impinged by compulsory voting is the weakest of all. The requirement to attend the polling booth once every three years violates no essential liberties. As Edward Mann said, individual liberty is freedom from unnecessary legal control; political liberty is participation in legal control. You can't have genuine individual liberty while refusing to participate in the system that regulates the society you live in. In fact, when a wide majority of people participate in the political process, it acts as a guard against invasions of individual liberty. Voting and participating is a public good; everyone benefits even if their candidate doesn't get elected. Largescale non-voting threatens our political liberties and can create dangerous inequality in our democracy.

foundationbriefs.com

Page 74 of 104

Sept/Oct 2013

Aff Counters: No Threat to Personal Liberty

Abstract compulsion is enough to spur turnout and political involvement, DAT 9/10 12:03am Malkopoulou, Anthoula. Lost Voters: Participation in EU Elections and the Case for Compulsory Voting. Center for European Policy Studies (July 2009).
In Cyprus and Luxembourg, the fines are higher, but there is little evidence of enforcement, with no single prosecution ever being reported in Luxembourg. Abstainers are asked to pay a maximum of CYP200 (approx. 342) (Election Law, 1979) and 1,000 respectively (Election Law, 2003). Among other reasons, nonpunishment results from the unpopular and disproportionately high fines and the cumbersome administration of their collection. In the latest discussion over the issue in 2008, the Cypriot Parliament decided to replace the judicial route of punishment with a direct and simple fine, in order to make prosecution more realisable (Politis, 2008). Despite this lax application, participation levels are extremely high in both countries; around 80 and 90% or higher. In Greece too, no enforcement system is in place. Yet the election law clearly stipulates sanctions for nonvoting that entail imprisonment or deprivation of public office (Constitution, 2001; Election Law, 2007). Although no implementation mechanisms are in place and compulsory voting exists only in abstracto, participation rates (around 80%) are still higher there than in the rest of the EU, the recent 53% being the lowest ever recorded. The same is true for Italy, which abolished the law in 1993. Hence, in most cases, even when enforcement is very lax or absent, the system seems to work. Surprisingly, voters still tend to adhere to the rule, not really for fear of punishment, but rather driven by popular belief or political custom. This comes down to how you can justify what compulsory voting really is (and opens a new thread of argument in doing so). While voting is coded into law in the aforementioned countries, the policy is not enforced. The idea of cost or penalty, seemingly inherent to the enforcement of compulsion, is no longer a factor, while the effects of compulsion with respect to voter turnout remain. The argument could be made for such a system against more practical neg assertions concerning enforcement and personal liberty.

foundationbriefs.com

Page 75 of 104

Sept/Oct 2013

Aff Counters: No Threat to Personal Liberty

The right to vote does not imply a right not to vote, DAT Hill, Lisa. Compulsory Voting in Australia: A Basis for a Best Practice Regime. Federal Law Review, vol. 32.
Another important consideration is to ensure that compelling citizens to vote does not interfere with any existing rights, particularly those that are constitutionally protected. In general, the question of a right to abstain has attracted little attention, most likely a function of the fact that, unless mandatory voting is being exploited as a means by which to forge consent (as was the case in single party states which enforced voting), few citizens are anxious to divest themselves of such an important and often hard-won right. Nevertheless, objectors in the United States might, for example, claim that compulsory voting limits political freedom and that the right to vote implies a right not to vote. Similarly, if Australia were to adopt a bill of rights, voting libertarians might argue that its protection of political freedoms conflicts with the requirement to vote. In fact, the existence of such a right has been tested legally and, so far, has been found to be non-existent. In the United States it has been indirectly challenged a number of times via constitutional challenges to voter purge statutes. Almost all have been unsuccessful including one that explicitly argued that voter purge statutes violate the right not to vote and thereby express political dissatisfaction. Similarly, in 1971, a case against compulsory voting was heard in the European Court of Human Rights under Article 9 of the European Convention on Human Rights ('freedom of thought, conscience and religion').47 In the case of X v Austria the Court ruled that, provided there was no compulsion to mark the ballot formally, compulsory voting does not violate the right to freedom of conscience and thought. (1)

Liberalism allows for civic duties. RMF Engelen, Bart. "Why liberals can favour compulsory attendance." Politics 29.3 (2009).
According to [Political Philosopher Anabelle] Lever, liberals cannot support compulsory voting because it promotes a view of what the good life consists of, namely a life of active political engagement. Liberals should design electoral systems in such a way that they respect reasonable disagreement among citizens on the value of political participation. However, liberals in favour of compulsory attendance do not refer to some comprehensive doctrine, according to which political participation is part of the good life and should therefore be made mandatory by law. Instead, all they claim is that voting is a civic duty that can be legally formalised in order to avoid free-riding problems. Precisely as a liberal state has the legitimate power to make its citizens pay taxes and serve in juries, it can stimulate its citizens to vote in democratic elections. Since it does not impose a specific conception of the good life that is not shared by all reasonable citizens, this is perfectly consistent with the liberal aim to design rules that enable members of a society to live together in mutual respect for each other as free and equal human beings. (220)

foundationbriefs.com

Page 76 of 104

Sept/Oct 2013

Aff Counters: No Threat to Personal Liberty

Liberty cannot be achieved in democracy without popular sovereignty. RMF Engelen, Bart. "Why liberals can favour compulsory attendance." Politics 29.3 (2009).
Both liberalism and democracy are ultimately grounded on and co-originate from the fundamental principle of mutual respect for each person as a free and equal human being. Without individual rights and liberties, democracies continuously face the threat of totalitarianism. However, without popular sovereignty guaranteed by a democracy in which people participate in the decisions that will bind them individual rights and liberties remain purely formal and empty. The basic insight that both elements are based on the same principle of moral and political autonomy is shared by Rousseau, Kant, Mill, Rawls and Habermas.

foundationbriefs.com

Page 77 of 104

Sept/Oct 2013

Aff Counters: Abstention Still Exists

Abstention still exists under compulsory voting


Blank ballots are common in countries with CV. RMF Lacroix, Justine. "A liberal defence of compulsory voting." Politics 27.3 (2007): 190-195.
However, compulsory voting does not in the least imply that deliberate abstention may not exist as exemplified by the high proportion of blank or null and void votes in countries where voting is compulsory. In Belgium (one of the four European countries where the vote is compulsory) the proportion of null and void votes was 7 per cent in the 1990s, against 1 per cent or less in most countries. One should not forget either that the expression compulsory voting or the duty to vote is a misnomer (Lijphart, 1997, p. 2). Strictly speaking, no one is compelled to vote. No one is obliged to fill in a ballot paper or to choose one of the parties or candidates in contention. The only duty that a citizen has to fulfil is to come to the polling station on a precise date. The citizens right not to vote is guaranteed by their presence in the polling booth and by the secret voting procedures. In that respect, it is significant to note that the European Court of Human Rights ruled that the obligation to vote did not go against article 9 of the European Convention for the Protection of Human Rights relative to freedom of thought, conscience and religion and to article 3 in the First Additional Protocol that establishes the right to free elections. In its ruling, the European Court declared that the citizens freedom of conscience was preserved because they could always cast a blank vote. (192-193)

In practice, all CV laws allow for casting a non-vote. RMF Engelen, Bart. "Why liberals can favour compulsory attendance." Politics 29.3 (2009).
A number of simple institutional measures can ensure that no citizen is ever obliged by the state to express his or her political views, even in countries with compulsory voting laws. In my view, these are required conditions that have to be fulfilled before a country can legitimately implement such laws. They guarantee that each and every citizen is allowed to think for him/herself, to speak for him/herself and, if he or she wishes, to remain silent. First, there is the secrecy of the ballot. As a matter of fact, compulsory voting is a misnomer. What is made compulsory is not voting, but attendance at the polling station. All a citizen has to do to comply is register his or her presence. The state has no control whatsoever over his or her choice inside the voting booth. As long as the ballot is secret, voting simply cannot be made compulsory. In this sense, it would be more accurate to speak of compulsory attendance or compulsory turnout. As Arendt Lijphart (1998, p. 10) perhaps the best-known proponent of compulsory attendance rightly argues, the secret ballot guarantees that the right not to vote remains intact. However, in each of the countries that implement such [compulsory voting] laws, the ballots secrecy is guaranteed. (218-219)

foundationbriefs.com

Page 78 of 104

Sept/Oct 2013

Aff Counters: Abstention Still Exists

Despite being able to opt out of voting, CV still remains important to turnout and attitudes. RMF Engelen, Bart. "Why liberals can favour compulsory attendance." Politics 29.3 (2009).
It may seem strange to defend compulsory attendance laws while stressing that their enforcement should not be strict. Nevertheless, empirical research shows that compulsory attendance laws, even when not actively enforced by means of harsh penalties, engender compliance. Turnout in countries with no enforcement is about 6 per cent higher than in countries with no compulsory voting (IDEA, 2002, p. 110). Here, formal laws are mainly a symbolic reminder that going out to vote is desirable. As such, they uphold the social norm and civic sense of duty to vote, which suggests that compulsory voting is a cultural rather than legal phenomenon (Hill, 2002, p. 95). In short, compulsory voting can ... be very effective in raising turnout in spite of low penalties that are imposed for failing to vote (usually similar to a parking violation), in spite of the lax enforcement (usually much less stringent than parking rules are enforced), and in spite of the secret ballot, which means that an actual vote cannot be compelled in the first place (Lijphart, 1998, p. 2). (219-220)

Abstention as a form of protest can be effectively carried out under CV, DAT Power, Timothy J., and J. Timmons Roberts. Compulsory Voting, Invalid Ballots, and Abstention in Brazil. Political Research Quarterly, Vol. 48, No. 4 (Dec., 1995).
Low voter turnout and the desire to strengthen democratic institutions have inspired many to propose that voting be made mandatory by law. In Brazil, as in Australia, Belgium, Italy, and a number of other countries, voting has in fact been compulsory for decades.' However, the case of Brazil alerts us that compelling voters to go to the polls does not guarantee that they will complete a ballot. While invalid votes in wealthy democracies such as Australia and the Netherlands have usually comprised about 2 to 3 percent of all ballots cast, in Brazil the equivalent rates are from five to twenty times higher. During Brazil's military regime of 1964-85, blank and spoiled ballots were often interpreted as a form of protest against the dictatorship. However in re- cent, democratically conducted elections in 1986 and 1990, rates of invalid voting set new records, skyrocketing to 40 percent of all ballots cast for the lower house of the National Congress. In addition, in 1990 some 15 percent of potential voters failed to show up at the polls altogether, most of these in violation of the law. (578)

foundationbriefs.com

Page 79 of 104

Sept/Oct 2013

Aff Counters: Abstention Still Exists

Other forms of protest do not have to be problematic, DAT Engelen, Bart. Why Compulsory Voting Can Enhance Democracy. Acta Politica, 2007, 42.
Some opponents of compulsory voting, however, contend that higher turnout levels are not necessarily a good thing. Compelling citizens to participate, even when they have no opinion or do not want to express it, only results in a higher number of protest votes (Keaney and Rogers, 2006, 30). The legitimacy of the democratic process is not enhanced if citizens cast their votes only because they have to. This argument is forceful in pointing out the possibility that their vote does not reflect their true preference (which is to stay at home) or opinion (which they often lack). If citizens freely decide to participate, their vote will be more authentic. To assess this argument one can analyze to what extent compulsory voting coincides with invalid, blank and other protest votes. The data show that the number of votes that do not count (invalid and blank ballots) drops about 2% when countries abolish compulsory voting. The amount of lost votes is thus substantially smaller than the amount of votes gained by compulsory voting. However, the problem lies in interpreting the group of protest votes that do count. Citizens who would not vote if they were not obliged to might vote at random. This phenomenon of donkey voting after the game in which a blindfolded child pins the tail on the donkey is not to be neglected. The problem is that the exact number of such votes is hard to measure. Additionally, they also occur in voluntary voting systems (Orr, 2002, 575). While pleading for more research in this respect, I want to refer to the options of leaving the ballot blank or spoiling it as an expression of ones protest. In my view, these have to be made more attractive, for example by adding a box with none of the above or providing space for personal comments (Hill, 2002a; Keaney and Rogers, 2006, 3132; Orr, 2002, 578). This way of collecting protest votes would partly remedy the failure of current compulsory voting systems to differentiate between politically, anti-politically and apolitically motivated protest voters. As protest votes are as valuable as other votes in a democracy, one should give them more electoral weight than they currently have. (28)

foundationbriefs.com

Page 80 of 104

Sept/Oct 2013

Aff Counters: Alternatives to Abstention

Better Alternatives to Abstention under Compulsory Voting Rules


The Right to Withdraw Democratic Legitimacy JMR Keaney, Emily and Ben Rogers. A Citizens Duty Voter Inequality and the Case for Compulsory Turnout. Institute for Public Policy Research. May 2006. Pg 30
Related to this argument is the belief that compulsory turnout takes away the power to withdraw democratic legitimacy from a government by not voting. However, as we argue below, a vital part of a compulsory turnout system would be some form of formalized protest vote. This would in fact be a far more effective means of withdrawing democratic legitimacy than abstention, as it could not be misread as apathy. Compulsory turnout implemented in this way can, therefore, be seen as strengthening our democratic processes.

CV Protects the Right to Say No JMR Levine, Jonathan. The Case for Compulsory Voting. The National Interest. Nov. 2012.
Any scheme for compulsory voting must unequivocally protect the right to say no. Nevada offers an interesting solution: in 1975, facing unprecedented voter disgust over Watergate, Nevada became the only state in America to include a none of these candidates option alongside the major parties. And while it was a step in the right direction, the initiative was nonbinding and toothless. In the 1976 Republican Congressional primary, none of these candidates trounced its competitors, and yet, despite the abjectly clear will of the voters, the nomination went to the nearest runner-up (who, perhaps unsurprisingly, went on to lose to the Democratic incumbent). For compulsory voting to be viable, the public must be given a full complement of options. All states should present voters with the option of none of these candidates, and should require the parties to field new candidates in another round, should that be the will of the people. This argument proposes that an even better alternative to abstention is that the ballot has an option none of these candidates for the voter to voice their discontent with the candidates. This option would require the democratic system to be pay more attention to the will of the people so that if the majority does not find any of the candidate suitable another round of candidates must be introduced. In practice, this method would strengthen the democratic process considerably more than simply allowing for abstention.

foundationbriefs.com

Page 81 of 104

Neg Counters

foundationbriefs.com

Page 82 of 104

Sept/Oct 2013

Neg Counters: Doesnt Increase Knowledge

CV does not increase political knowledge


Multiple studies have found no link between CV and increased political knowledge. RMF Loewen, Peter, Henry Milner, and Bruce Hicks. "Does Compulsory Voting Lead to More Informed and Engaged Citizens? An Experimental Test." Canada Research Chair in Electoral Studies, University of Montreal (2008). Web.
For example, Bilodeau and Blais (2005) could uncover no empirical studies to support Lipjharts claim. To fill the gap, they attempted to substantiate his claim in three ways. They first examined whether citizens in Western European countries with compulsory voting report that they discussed politics more than those in noncompulsory countries. Second, they examined the behaviour of immigrants to New Zealand from compulsoryvoting Australia. Third, they examined the behaviour of immigrants to Australia from compulsory voting countries. In each case they sought differences in reported levels of political discussion, interest in politics and attitudes toward voting, but were unable to find evidence of second-order effects due to compulsory voting. A recent analysis of Belgian survey data by Engelen and Hooghe (2007) could not find evidence of knowledge effects from compulsory voting. They used the hypothetical question what if voting were not compulsory to isolate those who vote to avoid sanction. They find evidence that those who vote to avoid sanction are less knowledgable about and engaged in politics, suggesting that while compulsory voting is effective at bringing the otherwise less engaged to the polls, it is not necessarily effective at increasing their knowledge levels. Another recent study using data from the Polish Election Survey used the same method in reverse, asking nonvoters what they would do if voting were compulsory (Czesnik 2007). Not surprisingly, those who reported voting to avoid sanction were the least interested and knowledgeable. As with the Belgian study, this merely demonstrates that compulsory voting would bring the otherwise less knowledgable to the polls. Finally, Ballinger (2007) looked at the British and Australian evidence, concluding that Australian respondents are no better-informed about political systems than British respondents. (4-5)

foundationbriefs.com

Page 83 of 104

Sept/Oct 2013

Neg Counters: Doesnt Increase Knowledge

Experimental tests on CV find no increase in knowledge or effort. RMF Loewen, Peter, Henry Milner, and Bruce Hicks. "Does Compulsory Voting Lead to More Informed and Engaged Citizens? An Experimental Test." Canada Research Chair in Electoral Studies, University of Montreal (2008). Web.
To test these hypotheses, we conducted an experiment among eligible-to-vote students at a Montreal CEGEP during the March 2007 provincial election. The logic of our experimental design is quite simple. We recruited a group of students to participate in a study about youth attitudes, consisting of two surveys administered approximately one-month apart, at either end of a provincial election campaign. All students who completed these surveys were eligible to receive $25 (CDN).2 However, to receive this money a randomly selected subset of the students were also required to vote in the provincial election.3 Accordingly, we were left with two groups, one of which faced a financial disincentive if they chose not to vote, the other of which faced no such disincentive. By comparing differences between these two groups in political knowledge, media news consumption and reported discussion about politics, we are able to draw inferences about the effects of compulsory voting-like incentives on voters, especially first-time voters. (6) We find little support in our data for the above hypotheses... As can be seen, the overall difference in knowledge scores in the second round between groups under treatment and control conditions is not significant. On average, both groups appear to be able to answer approximately four of ten political knowledge questions correctly. We next consider the possibility that the treatment students did try to learn more about politics but were unable to do so. We find no evidence that they increased their general engagement with politics through discussion, which could have signalled greater effort at learning. Rather, by the end of the campaign those in control and treatment both appeared to engage in conversation with friends and family somewhere between the Rarely and Sometimes response categories. (11)

foundationbriefs.com

Page 84 of 104

Sept/Oct 2013

Neg Counters: Doesnt Increase Knowledge

CV does not Increase Voter Interest, JMR Rovensky, Jan. Voting: A Citizens Right, or Duty? The Case against Compulsory Voting. Guido Carli Free International University for Social Studies. 2007. Pg 90.
The same amount of doubt is present in the assumption that compulsory voting will increase the interest (expressed by participation) of people in politics; though Engelen and Hooghe (2007: 13-5) support the institution of compulsory voting, their research shows that CV does not in the European context seem to produce a higher than average political interest, nor does it boost feelings of political efficacy. Turning yet again to the Australian context: Ballinger (2006: 14) cites data which clearly demonstrates that there is virtually no difference in the engagement, or rather disengagement, amongst young voters when comparing young Australians with young Britons. Obviously such a question would seem irrelevant in Australia where turnout is obligatory, however the cited research shows that if CV were to be abolished, only half of 16-18 year-olds would vote (as opposed to 87% if participation is compulsory) and overall turnout in Australia would fall to the region of 50-60% (Jackman 1999: 46) even after more than eighty years of compulsory voting; a loss of around 30% of voters would result in the abolishing of compulsory voting in Belgium (Hooghe and Pelleriaux 1998), Brazil or Venezuela (Power and Timmons Roberts 1995) in other words, CV does not create the voting norm or lastingly shape behaviour, as Hill and Louth (2004: 26) confidently claim though in the same article they concede that it were doubtful ... that the voting habit would stay with us without the strong incentive of law (Ibid: 9). The Netherlands experienced such noticeable declines (around 15% on average) after the abolition of compulsory voting laws in 1970, yet even nowadays turnout still remains around the 80 percentile mark, i.e. more than the average of some current CV countries.

foundationbriefs.com

Page 85 of 104

Sept/Oct 2013

Neg Counters: Doesnt Increase Knowledge

CV does not Increase Voter Knowledge JMR Rovensky, Jan. Voting: A Citizens Right, or Duty? The Case against Compulsory Voting. Guido Carli Free International University for Social Studies. 2007. Pg 90.
Apart from the two studies other evidence that compulsory voting promotes more political knowledge on behalf of the citizens is scarce Grnlund and Milner (2006) actually found that CV countries had an under-average political knowledge amongst their citizens. Schmidt (1974 quoted in Katz 1997) found that in the Netherlands the people who voted regularly before 1970 but failed to do so after the abolition of the compulsory vote, were disproportionately uninterested and uninformed about politics. Also Loewen, Milner and Hicks (2007: 12) after conducting a survey amongst young Canadian voters discovered that providing young voters with a financial disincentive from abstaining from voting did not increase how much they learned about politics. Though the aim of the authors is not to prove a point against compulsory voting (indeed, the opposite seems more accurate), in the end they are forced to concede that most arguments for the merits of compulsory voting would claim that is the effects are conditional, then they are most likely to manifest themselves among those who would otherwise be unengaged, particularly youth (Ibid: 16). In other words, if CV were to have an impact, it should be most felt amongst the younger generation; their research, however, proves that this is not the case with increasing political knowledge through a system of compulsion.

foundationbriefs.com

Page 86 of 104

Sept/Oct 2013

Neg Counters: No Cure for Apathy

There is no Cure to Apathy


Why More Voting is Not Always Good AMS Fred L. Smith. A Case against Mandatory Voting. American Spectator. http://spectator.org/archives/2011/07/25/case-against-mandatory-voting
Ornstein touted as a success the fact that, under Australias compulsory system, only about three percent of voters write in X. Meanwhile, in America, over 40 percent of eligible voters dont even go to the polls in any given election. So Australia is better off, right? If Ornsteins goal is to get only three percent of eligible American voters choosing none of the above, then he will also have to deal with 37 percent of uninformed, disinterested, and apathetic Americans being forced to cast ballots for candidates about whom they know little, if anything at all. What good could come of that?

Apathy is inherent to mass democratic voting systems, DAT Macedo, Stephen. Against Majoritarianism: Democratic Values and Institutional Design. Boston University Law Review, Vol. 90:1029.
Mass elections for legislatures or executives in constitutional democracies often empower governments that represent only pluralities of the public, rather than majorities. In these real world systems, moreover, the choices that voters face are typically highly constrained: a limited number of parties, perhaps only two, compete for our support. Moreover, the office of modern citizen is not such as to elicit a high level of responsibility. It is no insult to observe, with Schumpeter, that being one citizen among millions is like being a member of an enormously large committee the incentive to do ones homework before meetings is weak. Mass popular elections surely do as they ought matter greatly in modern representative systems, and legislative deliberation is often highly admirable, well-informed, conscientious, and dignified. But there is no insult or affront either to the voters or to the democratic principle of political equality if, in designing and adopting popular constitutions, additional mechanisms are adopted to further improve the quality of collective deliberation: expert administrative agencies, politically arms-length commissions, and courts with the power of review. Political institutions such as these are crucial features of modern democratic systems. They are accountable not on the basis of elections but often based on demanding expectations that reasons will be given in public and subjected to intense scrutiny. (1036-1037) Macedo essentially argues that safeguards of democracy can be found in institutions apart from voting. This is also a good counter, then, to aff arguments that center on voting as a central bastion of democracy.

foundationbriefs.com

Page 87 of 104

Sept/Oct 2013

Neg Counters: Consensus not a Dem. Value

Consensus is not a democratic value.


Politics Cant be Moderated AMS Fred L. Smith. A Case against Mandatory Voting. American Spectator. http://spectator.org/archives/2011/07/25/case-against-mandatory-voting
In his previous writings on the topic, Ornstein argues that mandatory voting will bring America to the center and eliminate the polarizing effect of partisan politics, especially in primary elections . His theory is that elective voting creates an environment where parties stir up their bases, leading to the election of increasingly more liberal Democrats and increasingly more conservative Republicans. With all of these radicals in office, he argues, valuable Congressional time is spent on frivolous or narrow issues (flag burning, same-sex marriage) that are intended only to spur on the party bases and ideological extremes. Consequently, important, complicated issues (pension and health-care reform) get short shrift. Who decides which issues are important? Shouldnt politicians respond to what their constituents tell them is important? I chided Ornstein for trying to make everyone play nice, as if politics could somehow lead to consensus through a utopian deliberative process. The Australian system he cites has not produced a placid political process by any means and its already been in use for decades.

True majoritarian rule and democratic legitimacy are not intertwined, DAT Berg, Chris. Informal Ballots: Blame Compulsory Voting. Institute of Public Affairs. 25 August 2010. Web.
There's another important argument against compulsory voting - we ought to have the freedom not to vote. In one of this country's few libertarian classics, Rip van Australia, John Singleton claimed it is the "ultimate contradiction for a supposedly free and democratic society to be founding on a system of compulsory voting." But Australia is a very utilitarian country. Arguments about rights and liberties don't get very far here. Many people claim that compulsory voting gives elected governments legitimacy. Put aside for a moment the implicit belief that the majority of democratic governments overseas are therefore somewhat illegitimate. If legitimacy is what we're seeking, then why not compel citizens to take turns running for parliament (like jury duty for Canberra) or insist they join a political party? Absurd, of course, but the legitimacy argument is too vague to be useful. (1)

foundationbriefs.com

Page 88 of 104

Sept/Oct 2013

Neg Counters: Consensus not a Dem. Value

Democracy does not require true majority rule, which is assumed under CV, DAT Macedo, Stephen. Against Majoritarianism: Democratic Values and Institutional Design. Boston University Law Review, Vol. 90:1029.
Ronald Dworkin seems to me correct with respect to a fundamental point which is very often missed: the basic principle of democracy is political equality3 and there is an important gap an interpretive gap between that principle and the more concrete rules and norms that structure decision making in particular institutional settings. Importantly, majority rule is not a fundamental principle of either democracy or fairness, nor is it required by any basic principle of democracy or fairness. Rather, it is one among a variety of decision rules that may, but need not, advance the project of collective legitimate self-rule based on political equality.4 Majority rule is a decision rule that has some nice properties, for example it is decisive when there are only two options, but its virtues, both practical and moral, are easily and frequently exaggerated. (1030)

foundationbriefs.com

Page 89 of 104

Sept/Oct 2013

Neg Counters: Response to Jury Duty

Response to Jury Duty Comparison


Jury Duty and Voting Duty Serve Different Purposes AMS Fred L. Smith. A Case against Mandatory Voting. American Spectator. http://spectator.org/archives/2011/07/25/case-against-mandatory-voting
The most common argument put forth by supporters of compulsory voting is that, just as Americans have a civic obligation to perform jury duty, they should face a similar obligation to vote. This seems like a tempting argument, but it is based on a fundamentally flawed understanding of the Constitution and the Bill of Rights. The Sixth and Seventh Amendments to the Constitution specify that both criminal and civil defendants have the right to a jury trial by their peers. For you to enjoy that right, your peers and you in turn must serve on a jury. This is one of the few instances where the Constitution compels citizens into service. There is no constitutional right to serve on the jury it is a constitutional requirement on all voting citizens. This compelled service is correctly called a civic duty. Voting is very different. No less than five constitutional amendments mention the right to vote, but nowhere in the Constitution is voting defined as a civic duty. As such, jury members are required to listen to both sides and then carefully deliberate before reaching a decision. Voters cannot be forced to listen to hours of campaign speeches before voting. Most importantly, jury trials and elections serve different purposes in the American system of government. Juries act as a check on the power of the state, by shifting some of the judicial decision-making power to private citizens. Voting, by contrast, is the process by which citizens delegate power to government. Therefore, compulsory voting would entail forcing large numbers of people to make an uninformed decision on a matter of crucial importance.

foundationbriefs.com

Page 90 of 104

Sept/Oct 2013

Neg Counters: None of the Above Not Enough

None of the above option not satisfactory


The act itself of not casting any ballot is needed, not just a none of the above option. RMF Lever, Annabelle. 2009. Is Compulsory Voting Justified?. Public Reason 1 (1): 57-74.
People can doubt the extent and reliability of their knowledge and judgement, or be unsure of the proper grounds on which to make their decision. They may feel that it would arbitrary and invidious to favour one of the candidates when several or all of them are acceptable and they may, of course, worry about the way that their vote will be interpreted and used by politicians and the media. So, even people who have no conscientious objections to voting might have compelling reasons to prefer abstention to voting in at least some elections, and to do so even if they have the option of voting for none of the above. (68)

foundationbriefs.com

Page 91 of 104

Sept/Oct 2013

Neg Counters: Other Options

There are Other Options


Practical Options AMS Anthony Morse. What is the Best Way to Increase Voter Participation Rates in the U.S.? Forbes. October 16, 2012. http://www.forbes.com/sites/quora/2012/10/16/what-is-the-best-way-toincrease-voter-participation-rates-in-the-united-states/
A few things we could do: Same day registration. If you arent enrolled, but come in to vote on election day and have ID of where you live, then you are allowed to vote. States have seen a three to twelve percent increase in voting. Move election day to Monday and make it a federal/bank holiday. Give everyone the day off and no other obligations for the day. People will have the time to vote, which is a big barrier to voting. Better civics education, especially around voting. All too often, I hear that my voting doesnt matter. Well it does, many local elections are determined by less than a thousand votes. Democracy is about figuring out who you like and who you dont and then convincing people who to vote for. Fix gerrymandering. Many districts are not competitive because both Democrats and Republicans screw around with congressional districts, state senate districts, and state house districts. This allows races to become non-competitive and discourages voting.

foundationbriefs.com

Page 92 of 104

Sept/Oct 2013

Neg Counters: Other Options

The Power of Social Pressure AMS Pacific Standard Magazine. Simple Ways to Increase Voter Turnout. March 31, 2008. http://www.psmag.com/politics/simple-ways-to-increase-voter-turnout-4660/
One experiment shows that just holding Election Day poll parties would notably increase turnout. But moresignificant results come from another experiment, in which a piece of direct mail informed voters that their participation was a matter of public record and that their neighbors would know whether or not they voted. This prick of social pressure increased voting rates by eight percentage points over the baseline rate, a finding that surprised even the professors behind it Yale political scientists Donald P. Green and Alan S. Gerber (University of Northern Iowa assistant professor of political science Christopher W. Larimer also worked on the experiment). After all, scholars and campaigns had already studied direct mail extensively and found that it didnt matter how colorful the mailing was or what it said nobody could find an effect of more than a percentage point. But they hadnt studied social pressure. We analyzed it for a week to see if there was some mistake or something was missing, Green said. Nothing changed. The experiment worked like this: During the 2006 Michigan primary elections, about 200,000 voters got no mailing. This control group voted at a rate of 29.7 percent. Then came the four mailings, each to 38,000 voters. Group 1 was told, Do your civic duty and vote! This increased turnout to 31.5 percent. Group 2 also got the civic duty reminder and then was told, You are being studied! (Members of Group 2 were informed that researchers would be watching them, though the results would remain confidential). This increased turnout to 32.2 percent. Group 3 got the civic duty reminder as well, plus information on whether they voted in the past election. They were also told, Who votes is public information. This increased turnout to 34.5 percent. Finally, and most notably, Group 4 received the civic duty reminder, plus a list of their neighbors voting histories all public information. They were asked: What if your neighbors knew whether you voted? This group voted at a rate of 37.8 percent almost 8 percent more than the control group. A good part of the explanation seems to lie in the power of social norms. Its possible that people simply felt that they were a little more attuned to their civic duty norm of participation once they had the sense they were compiling a track record, and I think that probably had a pretty substantial effect on their incentive, Gerber said, suggesting that these mailings might prick the civic conscience of a voter.

foundationbriefs.com

Page 93 of 104

Sept/Oct 2013

Neg Counters: Other Options

Give people a stake in their local government AMS Keith Wagstaff. Four Ways to boost the dismal turnout in Local Elections. The Week Magazine. September 2013. http://theweek.com/article/index/249385/4-ways-toboost-the-dismal-turnout-in-local-elections
Presidential campaigns often feature hot-button social and national security issues that drive voters to the polls. Local elections, on the other hand, center around the nitty-gritty details of governance, which is why they don't always inspire people to vote. Appropriating funds for parks and schools might be necessary, but it's not sexy. That is where participatory budgeting comes in. Based on a system created in Porto Alegre, Brazil, it gives citizens control over a slice of their city's budget. Starting last year in New York City, theParticipatory Budgeting Project convinced four city council districts to give their constituents power over $1 million of their budgets. Nearly 40 percent of the people who participated in the budgeting process said they rarely voted in local elections, according to The New York Times, which could make it a valuable tool to get otherwise apathetic voters to care about what happens in municipal government.

The Power of Peer Pressure AMS Keith Wagstaff. Four Ways to boost the dismal turnout in Local Elections. The Week Magazine. September 2013. http://theweek.com/article/index/249385/4-ways-toboost-the-dismal-turnout-in-local-elections
A Yale study found that participation in a 2006 Michigan election increased by almost 2 percent when voters simply got mail reminding them to go to the polls. Even more effective was including their neighbors' voting records with that mail a measure that boosted voter turnout by 8 percent. The study was repeated in local elections around the country with the same result. Showing people whether or not their neighbors voted in the last election "might prick the civic conscience of a voter," study co-author Alan S. Gerber told Pacific Standard. "It's possible that people simply felt that they were a little more attuned to their civic duty norm of participation once they had the sense they were compiling a track record, and I think that probably had a pretty substantial effect on their incentive."

foundationbriefs.com

Page 94 of 104

Sept/Oct 2013

Neg Counters: Other Options

Voting Ambience Helps AMS Keith Wagstaff. Four Ways to boost the dismal turnout in Local Elections. The Week Magazine. September 2013. http://theweek.com/article/index/249385/4-ways-toboost-the-dismal-turnout-in-local-elections
Aside from a sense of civic pride and a sticker, the rewards of voting can seem meager compared to the costs, which usually involve waiting inside a school gymnasium. It wasn't always like this. In the 1800s, "voters at the polls talked with friends, threw down shots of free whiskey, listened to lively entertainment, and generally had a good time," according to a 2006 study titled, "Putting the Party Back into Politics." Obviously, passing out shots of Jim Beam wouldn't fly today, but the Yale researchers found that throwing an Election Day poll party with "food, fun, and music" all complying with state and federal voter laws, of course increased voter turnout by 6.5 percent and cost less than direct-mail campaigns.

Modernize Voting Systems AMS Keith Wagstaff. Four Ways to boost the dismal turnout in Local Elections. The Week Magazine. September 2013. http://theweek.com/article/index/249385/4-ways-toboost-the-dismal-turnout-in-local-elections
Young people are notoriously unreliable voters. They are also fans of something called "the internet." Hence a push to use social media and apps to get younger voters to the polls. In Vancouver, city officials spent $10,000 on an app that would tell voters where and when to vote, who was on the ballot, and basic information about voter eligibility in the hopes of boosting turnout for a civic election. Last year, a study published in Nature found that a special non-partisan "get out the vote" message on Facebook consisting of photos of friends who had voted had "measurable if limited effect on voter turnout," according to The New York Times. Another option is online voting, something that has already been put in place in around 80 Canadian cities. In Markham, Ontario, overall turnout increased 10 percent in the four years after the city allowed its citizens to vote online. In the United States, concerns over voter fraud, denial-of-service attacks, and intrusions by foreign hackers has stymied the development of online voting. The country could follow New Zealand's example and use a handful local elections to determine whether online voting would work on a larger scale. With such low turnout already, some cities wouldn't have much to lose.

foundationbriefs.com

Page 95 of 104

Sept/Oct 2013

Neg Counters: Other Options

Success of Online Voting Systems AMS Canadian Press. Can internet voting boost turnout? CBC News. November 2011. http://www.cbc.ca/news/politics/story/2011/11/09/pol-cp-cyber-voiting-risk.html
Some of those who have adopted internet voting have seen a turnaround. In Huntsville one of 34 municipalities in Ontario that allowed e-voting last year voter turnout rose sharply. "Ours did go up from just around 30 per cent up to 46 per cent . . . and I feel that it will be even better the next time," says town clerk Kathleen Gilchrist. There were those in Huntsville who worried about hacking, but Gilchrist says the town chose a system operated by Halifax-based Intelivote Systems Inc., which had been previously tested for vulnerability in Vaughan, Ont. "They hired a company to see whether they could . . . break into the system and they couldn't." The vote in Huntsville went off with only a minor hitch one that didn't involve a security breach. The system was overloaded in the last hour of voting by candidates and their workers, who were monitoring activity. As a result, some voters were blocked and the voting deadline had to be extended by an hour to accommodate everyone. Intelivote has also handled political party leadership races such as the British Columbia Liberal vote last February and company president Dean Smith says while no system is perfect, they use encrypted data, security audits and other measures to prevent hacking.

foundationbriefs.com

Page 96 of 104

Sept/Oct 2013

Neg Counters: Other Options

Can Decrease the Frequency of Elections to positively Influence Turnout, JMR Lijphart, Arend. Unequal Participation: Democracys Unresolved Dilemma. The American Political Science review, Volume 91, Issue 1. March 1997. Pg 6.
The frequency of election has a strongly negative influence on turnout. Noyf (1981, 1986, 1989) has convincingly demonstrated this effect for the United States, in which he estimate that, on average, voters are asked to come to the polls between two and three times each year- much more often than in all except one other democracy. The one country with even more frequent dates on which election and referenda are conductedabout six or seven times per year-is Switzerland (Farago 1995, 121; Franklin 1996, 225, 234). The United States and Switzerland are also the two Western democracies with by far the lowest levels of turnout. The most plausible explanation is voter fatigue (Jackman and Miller 1995, 482-3) or, in terms of rational choice, the fact that frequent election increase the cost of voting. If frequent election depress turnout in first- order election, it is logical to expect that they hurt turnout in second-order elections even more. This may be the explanation for the wide gap in the United States between the first-order presidential election, on the one hand, and the secondorder- but in a system of separation and division of powers still very important- midterm congressional as well as state executive and legislative election on the other.

Automatic Registration Can Increase Voter Turnout JMR Lijphart, Arend. Unequal Participation: Democracys Unresolved Dilemma. The American Political Science review, Volume 91, Issue 1. March 1997. Pg 7.
Raymond E. Wolfinger and Steven J. Rosenstone (1980, 73, 88) found that turnout would increase by 9.1 percentage points if all states adopted completely liberalized registration rules, but they also argued that turnout would be raised substantially more by a European-style system in which registration is automatic or the governments responsibility. On the basis of his comparative analysis, Powell (1986, 36) concludes that automatic registration could boost turnout by up to 14 percentage points.

foundationbriefs.com

Page 97 of 104

Sept/Oct 2013

Neg Counters: No Help for Disadvantaged

Will not Help Disadvantaged Groups


The Disadvantaged Tend to Vote Against Their Own Interests AMS Jason Brennan. Should You be Forced to Vote? October, 2012. http://www.learnliberty.org/videos/should-you-be-forced-vote
One objection to this argument is that the disadvantaged, the poor, the unemployed, and the uneducated are less likely to vote than other groups. Some argue that people should be forced to vote so the disadvantaged wont be taken advantage of. Professor Brennan says this objection relies upon the false assumption that people vote for their own interests. In contrast, political scientists have found over and again that people tend to vote for what they believe to be the national interest. We dont need to worry about protecting nonvoters from selfish voters. Instead, we should worry about whether voters will invest the time to learn which policies really serve the public good. According to Brennan, bad decisions in the voting booth contribute to bad government; needless wars; homophobic, sexist, and racist legislation; lost prosperity; and more. While all citizens should have an equal right to vote, someone who wants to abstain from voting because he doesnt feel he knows the right answersor for any other reasonshould be allowed to do so. Brennan concludes that mandatory voting guarantees high turnout but not better government.

foundationbriefs.com

Page 98 of 104

Cases

foundationbriefs.com

Page 99 of 104

Sept/Oct 2013

Aff Case

Aff Case
Introduction:
Todays resolution questions how a government, specifically a democracy, conducts itself with respect to those it governs. Because such a government must do so justly, I value justice. The voting processthe democratic process as a wholeis best thought of as an equalizer. This is the tenet upon which democracies like the United States are builtone man, one vote. Such a system, in its ideal state, is inherently fair. Because such a system, in its ideal state would be considered fair, fairness ought to be valued as the guiding criterion for this debate. And because a voluntary vote system can be one conducive to an unfair apportionment of voting power between individuals, and potentially with more consequences an unfair apportionment of voting power between groups of people, I affirm the resolution that resolved: In a democracy, voting ought to be compulsory.

Contention One: Compulsory voting addresses the unfairness of socioeconomic divides effects on voting patterns
The framework through which to view voting equality is that of Jean-Jacques Rousseaus Social Contract, given both the presence of universal suffrage and the potential for representative democracy in his postulations (thus echoing the modern landscape). What Rousseau considered the general will was just thatthe will of all the people, with legislative implications for this, although this was different from the government, which deals with actual particular matters. The implication of Rousseaus political contract is that when the general will is acting on the direction of a state, it is generalit includes everyone, and can be considered general inherently due to its inclusivity; it is a fair representation of the governed. Contrast this with the realities of modern political processes. As noted by Annabelle Lever, the British have seen a 13% gap between manual and non-manual workers voter turnout by 2005; Arend Lijphart, in 1997, found this disparity to cross most other socioeconomic lines, with those of lower class generally voting less. The propensity to vote also decreases with youth. This is a problem. Even if these findings were reversed, it would still be a problem. Even under a values-centered debate, this is a problem. The reason for this is that the system itselfa voluntary voting systemallows such gaps to exist to begin with. Under the eye of any democratic government, the vote of someone wealthy does not hold more weight than the vote of a disadvantaged individual. The government represents both individuals equally. The government has jurisdiction over both individuals, and both groups to which those individuals belong. Thus, the problem with a voluntary voting system is that while the aforementioned assertions remain true, about individuals, they cannot any longer apply to groups. The difference between individual voter participation across socioeconomic groups equates to a difference in representation and governance across those socioeconomic groups (and across ages as well as just about any other demographics). This is in direct conflict with the idea of fairness in democratic governance, wherein a just government allows for every group to be both represented and governed in a proportionally

foundationbriefs.com

Page 100 of 104

Sept/Oct 2013 Aff Case consistent manner. For a government to govern in a just fashion, it must fulfill this responsibility. And because voting is essentially an expression of the general will in context of modern democracy, the process inherently has the necessity of being fair. This is best fulfilled under a compulsory voting system, which works to prevent the kind of marginalization that lends unfairness to the voluntary voting process.

Contention Two: Compulsory voting is an equalizer for voices of resistance


Inherent to many arguments against the requirement to vote is the idea that the right to vote is balanced by the right not to vote. Heres the problem (aside from the findings of Lisa Hill that the right not to vote isnt hardcoded as law) with this idea: the right not to vote is only a positive element if there is something inherently and uniquely beneficial to it. But is there? While not casting a vote is seen as a vote against whatever candidates are currently up for election, this is still possible under two forms with compulsory voting: 1) Not casting a ballot, or 2) Casting a blank or invalid ballot. This first option wouldnt seem possible under compulsory voting, but compulsion does not necessarily refer to the requirement to fill out a ballot; rather, it can simply refer to having to show up to a polling place. More profoundly, however, the second option is actually one that can more powerfully voice a peoples opinion and give them a clear voice in a democracy (which really is what a just democracy ought to be doing). Consider the case of Brazil, under whichas noted by Timothy J. Power casting invalid ballots was an active form of protest against the military regime. Contrast this with voter abstention in voluntary systems, which has done little other than spur further debate about how to increase voter turnout. One of these two outcomes clearly does a more active job encouraging fair treatment by the government of the populace, which is integral to its legitimacy. This again goes back to notions of equality and the general will. A non-vote can be seen as a sign of protest, but that non-vote action does not carry weight because in a democratic system, only a vote carries legitimacy because it is counted. As such, for any act of resistance or protest to be valued equally to a standard vote for a candidate, it must also be in the form of a vote. To make voting compulsory and allow for resistance within the framework of compulsory voting would make protest a valid and equal way to show opinion, rather one that is nominal (as under voluntary voting).

Contention Three: People deserve a fair choice


As many neg cases will note, the right not to vote can be important and wise if no candidate suits ones fancy. But a major reason this scenario repeatedly plays out is due to voluntary voting. The problem with any kind of voluntary voting system is that because people have the option of not voting and spending their time elsewhere, much of the battle becomes about getting people to vote. And as Norman Ornstein of the New York Times elucidates, that means rabid partisanshipdivisiveness played out fiercely on a political stage. That results in an unjustly broken paradigm facing democratic states: a populace that is largely moderate asked to select between extremist candidates. In being conducive to such a reality, voluntary voting systems artificially create an untenable and unfair set of options for voters, who must live in a reality divergent from their political inclinations. The fairness of an election is contingent upon the idea that people are given a fair choice. If a candidate is elected without voters having a fair choice in who is elected, the government loses legitimacythe

foundationbriefs.com

Page 101 of 104

Sept/Oct 2013 Aff Case general will is divergent from the legislative actions taken by the democracy. A government not attuned to the peoples will cannot govern justly, but this is exactly what happens when people are presented with an option set divergent from personal values. In encouraging a focus on issues (rather than turnout), compulsory voting creates a more fair paradigm in which candidates have a far greater potential for matching up with the will of the people. A government consisting of such elected candidates is far more likely to govern justly, given that it is a more precise reflection of the peoples will.

Conclusion
We trust the state to uphold ideas of justice. But voluntary voting systems in democratic states manage to subvert the foundations of such a state. For a democracy to be justto fulfill its core objectivemeans an emphasis on fairnesssomething that is sorely lacking without compulsory voting. A requirement to vote is not a burden. It is an enablerof socioeconomic representation and equality, of resistance, and of electoral ideals. In essence, it is an enabler of a fair electoral system, and by extension, a just government.

foundationbriefs.com

Page 102 of 104

Sept/Oct 2013

Neg Case

Neg Case
Introduction:
While many praise the freedom to vote for ones leaders, the freedom to refuse to vote for ones leaders is equally important in a democracy. If a citizen finds all candidates lacking, he is guaranteed the right to refuse a vote. Not only does the principle of compulsory voting systems contradict democratic values, it is entirely ineffective. My core value is utilitarianism, under the criterion of John Stuart Mills liberty principle, stating that the only purpose for which power can be rightfully exercised over any member of a civilized community, against his will, is to prevent harm to others. Because compulsory voting is not only an unjust exercise of power, but a direct cause of harm to the members of a democratic society, I negate todays resolution that Resolved: In a democracy, voting ought to be compulsory.

Contention One: Compulsory voting does not increase voter knowledge.


The affirmative argues that making voting mandatory will essentially force the members of a democratic nation to become politically active and learn about voting issues. In reality, making voting compulsory just increases the number of apathetic voters. According to a report from the Canada Research Chair in Electoral Studies at the University of Montreal, a 2007 study on the effects of compulsory voting laws showed that voters who vote to avoid sanction do not have increased knowledge levels as a result of mandatory voting. A study from the polish Election Survey similarly determined that voters who reported voting to avoid sanctions were the least interested and knowledgeable. A final third study examined British and Australian voters to conclude that Australian voters, who are legally compelled to vote, are no more knowledgeable about politics than British respondents. John Stuart Mills liberty principle holds that the only just uses of power in government are those that directly prevent harm to another person. Forcing apathetic members of a society does not improve the societyit only brings ignorant citizens into the voting pool, thereby violating the liberty principle, and failing according to my core value.

foundationbriefs.com

Page 103 of 104

Sept/Oct 2013

Neg Case

Contention Two: People who are forced to vote tend to vote against their own interests.
In addition to, or rather, because compulsory voting systems encourage ignorant voting, these voters tend to vote against their own interests. According to the European Journal of Political Research, unwilling or ignorant voters forced to the polls by CV tend to make choices that are considerably less consistent with their policy preferences than voluntary voters. Georgetown Universitys Ethics, Economics, and Public Policy Professor Jason Brennan explains that the idea that mandatory voting decreases inequality relies upon the false assumption that people vote for their own interests. In contrast, political scientists have found over and again that people tend to vote for what they believe to be the national interest. This shows that mandatory voting is not only ineffective, it is contradictoryleading to worse voting systems rather than improved ones and thereby again contradicting Stuart Mills criterion for utilitarian government.

Contention Three: Alternative methods of increasing voter turnout exist.


Anthony Morse of Forbes Magazine explains several untried methods of increasing voter participation. First, states in the U.S.A. that allow same day registration at elections have seen a 3 to 12 percent increase in voter turnout. Morse also suggests turning every Election Day into a federal holiday to allow citizens the time to vote. Other suggestions include fixing the flaws in the gerrymandering system to make all races competitive and encourage voter participation and even making poll parties more fun by supplying food to voters. Clearly, there are many methods of increasing voter turnout today that have not yet been tried. Turning to compulsory voting before examining these options is foolish and ineffective. The only just exercises of democratic power are those that prevent harm to citizens. Making voting compulsory fails this test because it will cause probable harm to citizens by bringing uninformed uninterested citizens into the voting populace. Thus, this resolution falls according to my core value.

Conclusion
Compulsory voting defies the values of a democratic system. It takes away the liberty of choosing to vote and replaces it with a government-sanctioned chore. Nations that have implemented such a system have seen a huge increase in ignorant voters, who tend to vote against their own interests. Instead governments must pursue less extreme ways of increasing turnout to uphold John Stuart Mills utilitarian liberty principle.

foundationbriefs.com

Page 104 of 104

Você também pode gostar